MBE KAPLAN--REAL PROPERTY Flashcards
A farm and an orchard are adjoining tracts of land located in a county. In 2006, a farmer purchased the farm, a 10-acre tract, in fee simple absolute. The orchard, a 20-acre tract situated to the north of the farm, was owned by a rancher in fee simple absolute. A remote predecessor of the farmer had granted to a shepherd a way for egress and ingress across the farm under such terms and circumstances that an easement appurtenant to the orchard was created. This right-of-way was executed by deed and properly recorded. The shepherd, however, never made any actual use of the right-of-way.
In 2010, the rancher conveyed the orchard to the farmer. The next year, the farmer conveyed the orchard by deed to an investor for a consideration of $250,000, receipt of which was acknowledged. Neither the rancher—farmer deed nor the farmer— investor deed contained any reference to the easement for right-of-way. The investor has now claimed that she has a right-of-way across the farm. The farmer, on the other hand, has informed the investor that no such easement exists.
Assume that both the farm and the orchard abut a public highway and that ingress and egress are afforded the investor by that highway. In an appropriate action by the investor to determine her right to use the right-of-way across the farm, she should
(A) lose, because the easement was extinguished by merger when the farmer acquired the orchard from the rancher.
(B) lose, because the right-of-way was abandoned inasmuch as there never was any actual use made.
(C) win, because the farmer had constructive notice of the easement.
(D) win, because the investor acquired an easement by implication.
- (A) This Multistate questions deals with extinguishment of on easement by merger. When an easement appurtenant exists and both the dominant and servient tenements come under the ownership of the same person, the easement is terminated by operation of law. The apparent rationale is that one cannot have an easement in his own property. Therefore, when the farmer (who was already the owner of the farm) acquired the orchard from the rancher, the easement in effect merged in his fee simple ownership of both properties. Choice (B) is incorrect. When an easement is extinguished or terminated due to abandonment, there needs to be a clear showing by the dominant tenant that s/he intends to abandon the use. This is evidenced by the conduct of the dominant tenant. Non-use coupled with an intent to abandon will generally suffice to show affirmative abandonment by the dominant tenant. Here, there was no such abandonment shown by the shepherd, who was the original dominant tenant. The shepherd’s lack of use was not enough to create an abandonment of the easement. Choice (B) is incorrect because the reason why the easement was lost was due to the merger of the dominant and servient estates when the rancher sold the orchard to the farmer. Choice (C) is incorrect. While it is true that the shepherd had recorded the right-of-way in the past (thus putting the farmer on constructive notice of it), that does not change the fact that the easement was still lost by merger. Because the servient and dominant tenements had merged due to the rancher’s conveyance to the farmer, the easement was lost despite the fact that there was a recordation of it. Because this choice focuses only on the recordation without taking into account the merger of the estates, it is incorrect. Choice (D) is incorrect. An easement may also be created, though not expressly in a writing, by implication. An easement by implication generally arises when the owner of two or more adjacent parcels sells one or more of them and it is clear (although no easement was mentioned in the instrument of conveyance) that one was intended. In order to establish an easement by implication, one of the requirements is that the easement be reasonably or strictly necessary to the use and enjoyment of the quasi- dominant estate. Even though the farmer had gained common ownership and sold the orchard portion to the investor (which would be the quasi-dominant estate), the highway provides sufficient ingress and egress such that the easement across the farm portion is neither reasonably nor strictly necessary. Thus, the investor did not acquire an easement by implication when the farmer conveyed the northern half to her.
- An orange grove and a vineyard are adjoining tracts of land located in a county. In 2006, a farmer purchased the orange grove, a 10-acre tract, in fee simple absolute. The vineyard, a 20-acre tract situated to the north of the orange grove, was owned by a rancher in fee simple absolute. A remote predecessor of the farmer had granted to a shepherd a way for egress and ingress across the orange grove under such terms and circumstances that an easement appurtenant to the vineyard was created. This right-of-way was executed by deed and properly recorded. The shepherd, however, never made any actual use of the right-of-way.
In 2010, the rancher conveyed the vineyard to the farmer. The next year, the farmer conveyed the vineyard by deed to an investor for a consideration of $200,000, receipt of which was acknowledged. Neither the rancher—farmer deed nor the farmer— investor deed contained any reference to the easement for right-of-way. The investor has now claimed that she has a right-of-way across the orange grove. The farmer, on the other hand, has informed the investor that no such easement exists.
Assume that the orange grove abuts a public highway. The only access to that public highway for the vineyard is across the orange grove. If the investor initiates an appropriate action to determine her right to use the right-of-way across the orange grove, she should
(A) prevail, because an easement by implication arose from the farmer’s conveyance to the investor.
(B) prevail, because once an easement is properly recorded, it remains in effect ad finem until expressly released.
(C) not prevail, because any easements were extinguished by merger when the farmer acquired a fee simple in the vineyard.
(D) not prevail, because the deed of conveyance from the farmer to the investor failed to contain any mention of the right-of-way.
- (A) This is an example of an easement by implication. An implied easement is created and proved not by the words of the conveyance (because there is no such express language), but by all the circumstances surrounding the execution of the conveyance. It is based upon the intent of the parties. There are five prerequisites to the creation of an implied easement: (1) There must be a quasi-easement; (2) there must be a conveyance of one part of the property and a retention of another part of the property by the grantor; (3) the quasi-easement must be apparent at the time of the easement; (4) the quasi-servient tenement must be permanently adapted to serve the quasi-dominant tenement; and (5) the quasi-easement must be (a) reasonably necessary to the enjoyment of the quasi-dominant tenement if that tract is conveyed, and (b) strictly necessary if the quasi-dominant tenement is retained by the grantor. Therefore, because the investor’s only access to the highway was across the orange grove, she would have an easement by implication. Choice (B) is incorrect. This choice suggests that because the easement was recorded by the shepherd, that it remains in effect indefinitely until it is expressly released. However, an easement can be extinguished by merger, and that is what occurred when the rancher conveyed the vineyard to the farmer. Because this choice focuses too much on only one way that an easement can be extinguished (by express release), it is too limiting and therefore incorrect. Choice (C) is incorrect. It is true that the easement granted to the shepherd was extinguished by merger when the farmer acquired a fee simpLe in the vineyard, thus extinguishing that easement given to the shepherd by express grant. However, when the farmer sold the former the vineyard portion to the investor and created a parcel that had no access to the public highway, an implied easement arose due to strict necessity. Because this choice ignores the creation of an easement by implication, it is incorrect. Choice (D) is incorrect. An easement can be created by express grant and can be mentioned in a deed of conveyance. That is but one way in which an easement can be created. Even though the deed faiLed to contain any mention of a right-of-way, an easement by implication may stiLl arise. Because this choice fails to account for other ways in which an easement can be created, it is incorrect.
- A landowner, being owner in fee simple of a farm conveyed the property by warranty deed to an investor. The investor gave a farmer a mortgage on the farm to secure a loan from the farmer to the investor in the amount of $50,000. The mortgage was recorded immediately. Two years later, the investor conveyed the farm to his friend by quitclaim deed. According to the deed instrument, the friend “assumed the mortgage.” The investor then defaulted on the mortgage, and the farmer brought an in personam action against the friend to recover the amount of the mortgage due.
The farmer will most likely
(A) prevail, because the friend acquired title to the farm by quitclaim deed.
(B) prevail, because the farmer was a third-party beneficiary under the conveyance between the investor and the friend.
(C) not prevail, because the conveyance between the investor and the friend did not effectuate a delegation of duties.
(D) not prevail, unless the land was subject to the mortgage.
- (B) A key testing area on the MuLtistate deals with mortgages. The difference between a transfer of the mortgage property “subject to” and “assuming” the mortgage is the difference between personal liability and the lack thereof on behalf of the grantee. In a conveyance of Land “subject to” a mortgage, the grantee is not personally liable for the debt that the mortgage secures. ConverseLy, in a sale of land in which the purchaser “assumes the mortgage,” the purchaser or grantee Is personally liable for the mortgage debt. Because the friend “assumed the mortgage,” he is personally liable; therefore, choice (B) is correct. Choice (A) is incorrect. A quitclaim deed is a deed that contains none of the six covenants of title. By quitclaiming title to the land, the grantor only transfers whatever title s/he holds in the land, but gives no covenants. The acquisition of title by quitclaim deed does not address the issue of whether or not the friend assumed any personal liability for the mortgage on the property. Because this choice does not address the friend’s liability and focuses only on deed covenants, it is incorrect. Choice (C) is incorrect. The friend did assume the mortgage underthe facts. That alone would make the friend personally responsible for the mortgage debt if the investor defaulted on the mortgage. An actual delegation of duties is not required because the deed itself stated that the friend “assumed the mortgage.” Because this answer choice falsely suggests that a delegation of duties was necessary in order to make the friend liable, it is incorrect. Choice (D) is incorrect. This choice suggests that the farmer will only prevail if the land was subject to the mortgage. However, the friend’s assuming of the mortgage creates liability and will provide a means by which the farmer can prevail. Because this answer choice fails to take into account the friend’s assuming of the mortgage, it is incorrect.
- A rancher, being owner in fee simple
conveyed the property by warranty deed to a woman. The woman gave her niece a mortgage on the ranch to secure a loan from the niece to the woman in the amount of $500,000. The mortgage was recorded immediately. Two years later, the woman conveyed the ranch to a farmer by quitclaim deed. The woman then defaulted on the mortgage, and the niece brought an in personam action against the farmer to recover the amount of the mortgage due.
Assume that the woman’s quitclaim deed to the farmer made no reference to the mortgage. The woman then defaulted on the mortgage, and the niece brought an in personam action against the farmer to recover the amount of the mortgage due.
The mortgagee will probably
(A) succeed, because an implied delegation of duties resulted from the woman’s conveyance to the farmer.
(B) succeed, because the niece was a third-party beneficiary under the conveyance between the woman and the farmer.
(C) not succeed, because the farmer did not promise to pay the mortgage debt.
(D) not succeed, unless the farmer had constructive notice of the existence of the mortgage.
- (C) In order for the grantee-farmer to be personally liable to the mortgagee, the farmer must “assume the mortgage.” Thus, in this situation where the deed instrument makes no reference to the mortgage, the grantee does not assume the mortgage and is not personally liable to the mortgagee. Choice (A) is incorrect. Where the deed language is silent, the grantee is considered to take “subject to” the mortgage. This means that the grantee is not personally liabLe for the mortgage debt. A deed’s silence on this matter does not imply a delegation of duties such that the farmer must now pay just because she was the woman’s grantee. An affirmative assumption of the mortgage must be stated in the deed and no delegation of the duty to pay will be implied by the conveyance. Choice (B) is incorrect. Because the deed was silent, the farmer took the property “subject to” the mortgage. Because the farmer took the property in this fashion, there is no agreement between the farmer and the woman that the farmer would pay the mortgage for the benefit of the niece. Thus, there is no third.party beneficiary situation like there was in Question 3. Choice (D) is incorrect. This choice suggests that if the farmer had constructive notice of the existence of the mortgage, then that alone would have made her liable for the debt. Even though the mortgage was recorded, that will not be enough to make the farmer liable for the mortgage debt because the woman—farmer deed never affirmatively stated that the farmer would assume the mortgage.
- For many months, a buyer had been negotiating with a man for the purchase of a tract of land. Finally, on August 18, 2009, the buyer and the man entered into a real estate sales contract that provided in part:
“I, (the man), agree to convey good and marketable title to (the buyer) 60 days from the date of this contract.” The stated purchase price for the tract was
$175,000.
On October 11, 2009, the buyer phoned the man and told him that his title search indicated that a third party, not the man, was the owner of record of the property. The man responded that, notwithstanding the state of the record, he had been in adverse possession for 21 years. The statutory period of adverse possession in this jurisdiction is 20 years.
The next day the buyer conducted an investigation that revealed that the man had, in fact, been in adverse possession of the tract for 21 years. At the time set for closing, the man tendered a deed in the form agreed in the sales contract. The buyer, however, refused to pay the purchase price or take possession of the tract because of the man’s inability to convey “good and marketable title.”
In an appropriate action by the man against the buyer for specific performance, the vendor will
(A) prevail, because he has obtained “good and marketable title” by adverse possession.
(B) prevail, because the man’s action for specific performance is an action in rem to which the third party is not a necessary party.
(C) not prevail, because an adverse possessor takes title subject to an equitable lien from the dispossessed owner.
(D) not prevail, because the buyer cannot be required to buy a lawsuit even if the probability is great that the buyer would prevail against the man.
- (D) Title established through adverse possession is free from encumbrance and of a character to assure quiet and peaceful enjoyment of the property by the vendee. However, it is not a “marketable” title of record until there has been a judicial determination of such title. To show a record title by adverse possession requires a suit and the recording of a decree. Even though a court may determine that the vendor had title by adverse possession, the vendee did not bargain for that kind of title when the contract required a “marketable” title of record. Therefore, in accordance with the prevailing view, choice (D) is correct. This specific problem is referred to in Smith and Boyer, Law of Property, pg, 264. Choice (A) is in correct. The vendor! seller will not have “marketable” title of record untilthere has been a judicial determination of such title. Until that happens, the seller does not have the “good and marketable title” as recited in the original sales contract. Choice (B) is incorrect. The man is suing the buyer in an in personam action because the man is asking the buyer to tender the purchase price. This choice is also incorrect in that it fails to address the fact that without a judicial decree, the man did not have marketable title of record and cannot prevail in his action against the buyer. Choice (C) is incorrect. An equitable lien is a charge placed upon property that is imposed by law to prevent unjust enrichment. This choice suggests that the dispossessed owner would be able to place an equitable lien on the property that the man (as the would-be adverse possessor) is taking by adverse possession. However, if title were vested in the man by a judicial decree, the man would take title free from encumbrances. In other words, if the man were to prevail, the actual owner would not be able to have an equitable lien placed on the property.
- An uncle owns his property in fee simple. The uncle executes and delivers a deed to the property with the following granting clause:
“To my nephew and my niece for the life of my nephew, then to my gardener.”
As a result of the grant, which of the following is the most accurate statement regarding the ownership rights of the property?
(A) The nephew and niece are joint tenants in fee simple defeasible, and the gardener has an executory interest.
(B) The nephew has a life estate, the niece has a life estate pur autre vie, and the gardener has a vested remainder.
(C) The nephew and niece are tenants in common for the life of the first of them to die, the nephew and niece have contingent remainders in fee simple defeasible, and the gardener has an executory interest.
(D) The nephew has a life estate, the niece also has a life estate, and the gardener has a vested remainder.
- (B) First of all, the conveyance created a life estate in the nephew, and the niece had a life estate pur autre vie for the life of the nephew. The gardener received a vested remainder in fee simple absolute. A vested remainder is a remainder created in an ascertained and existing person that is not subject to any condition precedent except the normal termination of the preceding estate. Because the gardener would take as soon as the nephew died, the gardener received a vested remainder. Choice (A) is incorrect. A fee simple defeasible is a fee simple interest that could last for an infinite duration but can be terminated upon the happening of a specific event. The nephew and niece had, at most, a life estate in the property. Moreover, the gar. dener did not have an executory interest; the gardener received a vested remainder. Choice (C) is incorrect. The nephew and niece did not receive contingent remainders in the property. In fact, they received no future interest. The nephew and niece received life estates, which are a present possessory interest in Land that are to take effect immediateLy. Moreover, this answer choice is incorrect because the gardener received a vested remainder upon the death of the nephew. The gardener did not receive an executory interest. Choice (D) is incorrect. As seen in the other answer explanations, the niece has a life estate purautre vie, not a life estate measured by her own life.
- A woman was the fee simple owner of a 20-acre tract of land. When the woman moved to another state, a man took possession of the tract. The man’s possession has at all times complied with the requirements of the applicable adverse possession statute in effect.
Twelve years after the man took possession, the woman died intestate, leaving her six-year-old son as her only surviving heir. Nine years after the woman’s death, when the son was 15, the son’s guardian discovered that the man was in possession of the tract. Assume that the statutory period of adverse possession is 20 years and the age of majority is 18.
Which of the following correctly describes the state of title to the tract?
(A) The man has acquired title by adverse possession.
(B) The man will not acquire title unless he continues in adverse possession for an additional three years, or until the son reaches the age of 18.
(C) The man will not acquire title unless he continues in adverse possession for an additional eight years, making a total of 12 years after the woman’s death.
(D) The man will not acquire title unless he continues in adverse possession for an additional 12
years, or nine years after the son attains the age
of 18.
- (A) In this adverse possession problem, the facts indicate that the statutory period of adverse possession in this jurisdiction is 20 years. Looking back, the facts tell you that 12 years after the man took possession (of the tract), the woman died. Next, nine years after her death, the son’s guardian discovered the man in (adverse) possession of the tract. Now, adding the 12 years (before the woman’s death) to the nine years (following death) would create 21 years. Thus, because the man’s adverse possession is beyond the statutory period, choice (A) is correct. Choice (B) is incorrect . The disability of the successor, the son, would have no effect because the man is already in possession of the tract due to his fulfillment of the statutory requirements. Therefore, the man does not have to continue on for an additional three years. Choice (C) is incorrect. The disability of the successor, the son, would have no effect because the man is already in possession due to his fulfillment of the statutory requirements. Therefore, the man does not have to continue on for an additional eight years. Choice (D) is incorrect. Because the disability of the successor, the son, would have no effect here, the man is already in possession of the tract due to his fulfillment of the statutory requirements. Therefore, the man does not have to continue on for an additional 12 years.
- A sister and brother own a ranch as joint tenants. If the sister transfers her interest by quitclaim deed to her friend without the brother’s knowledge, what interest, if any, does the friend have in the ranch?
(A) No interest.
(B) An undivided one-half interest with right of survivorship.
(C) An undivided one-half interest without right of survivorship.
(D) A lien against the entire property.
- (C) If one joint tenant conveys his interest in the property to another, even if the conveyance is done secretly, severance occurs, whereby the right of survivorship is severed and a tenancy in common results. Based on the facts presented, the friend would take an undivided one-half interest in the ranch without right of survivorship. Choice (A) is incorrect. When the sister conveyed to her friend by quitclaim deed, she transferred to the friend whatever interest she happened to own at the time. The sister’s unilateral severance of the joint tenancy does not change the fact that she still transferred some interest in the ranch to her friend. Choice (B) is incorrect. When the sister (as a joint tenant) conveyed the property to the friend, that act alone severed the joint tenancy and destroyed any right of survivorship that once existed. The friend will take an undivided one-half interest, but it will not be with any right of survivorship. Choice (D) is incorrect. The friend was the sister’s grantee and owns an undivided one-half interest in the ranch. The friend does not have a lien against the property because he is not a creditor.
- A wife and husband are married and own a dairy as tenants by the entirety. If the wife transfers her interest in the dairy by quitclaim deed to her aunt without the husband’s knowledge, what interest, if any, does the aunt have?
(A) No interest.
(B) An undivided one-half interest with right of survivorship.
(C) An undivided one-half interest without right of survivorship.
(D) A lien against the entire property.
- (A) A tenancy by the entirety, unlike a joint tenancy, does not allow either party to convey away his or her interest in the property without the other’s consent. As a result, the aunt would not have any interest in the dairy. Choice (B) is incorrect. With a tenancy by the entirety, neither spouse can individually dispose of any interest in the estate; rather, both must join in the conveyance. Because of this, the aunt does not have an undivided one-half interest. Choice (C) is incorrect. With a tenancy by the entirety, neither spouse can individualLy dispose of any interest in the estate; rather, both must join in the conveyance. Because of this, the aunt does nothave an undivided one-half interest. Choice (D) is incorrect. With a tenancy by the entirety, neither spouse can individually dispose of any interest in the estate; rather, both must join in the conveyance. Because of this, the aunt does not have a lien or any other interest in the dairy.
- A man, his nephew, and his son are the owners of three contiguous lots in the city. A downward slope exists from the man’s land to the son’s land.
The man’s and the nephew’s lots were in an unimproved natural state. The son, however, had lived for 10 years in a house that he had built on his property.
In 2009, the man planted trees and shrubbery on his land along the boundary of the nephew’s lot.
In 2010, the nephew, in preparation for building a house on his lot, carefully excavated an area eight feet deep for the purpose of building a basement. The side of the excavation closest to the man— nephew boundary suddenly collapsed, and a quantity of the man’s soil, trees, and shrubbery fell into the hole. The nephew hauled away the debris.
In an appropriate action by the man against the nephew to recover for the damage to his land, judgment should be for whom?
(A) The nephew, if he was conducting the excavation work non-negligently.
(B) The nephew, because he was under no duty to support the man’s land in its improved state.
(C) The man, because a landowner is entitled to support of his land in its natural condition.
(D) The man, because a landowner has an absolute right to have his land supported by the neighboring land.
- (C) One who by excavation, or other methods, withdraws lateral support from her neighbor’s land is liable for the injury done to such land in its natural condition, regardless of negligence. The common law right to lateral support is a right to the support of land in substantially its natural condition. It does not include the right to have the additional weight of artificial structures supported by the neighboring land. However, a defendant may be held liable for damage caused to structures if there is proof of negligence on his part. In the present example, the man’s land was essentially in its natural condition. As a result, choice (C) states the correct rule. Choice (A) is incorrect. Because the man’s land was in a natural condition, he has an absolute right to lateral support. So when the nephew withdrew lateral support from his neighbor’s land, the nephew became absolutelyandstrictly liable (regardless of negligence) for the damage to the man’s land. Because this choice falsely suggests that lack of negLigence would somehow be a defense to the nephew in this situation, it is incorrect. Choice (B) is incorrect. Because the man’s land was in a natural condition, he has an absolute right to lateral support. So, when the nephew withdrew lateral support from his neighbor’s land, the nephew had an absolute duty to the man not to cause any damage to the land from a withdrawal of lateral support. Choice (D) is incorrect. Although this choice is correct in stating that the man had an absolute right to have his land supported, it fails to explain why the man has that right in this situation. It is only because the land was in its natural condition with no artificial structures on it that the man has this absolute right to lateral support. Because choice (C) is more complete in giving the reason why the man is entitled to this right (the land stayed in its natural condition), it is preferred over choice (D).
- A homeowner, his daughter, and his sister are the owners of three contiguous lots in the city. A downward slope exists from the homeowner’s land to the sister’s land.
The homeowner’s and daughter’s lots were in an unimproved natural state. The sister, however, had lived for 10 years in a house that she had built on her property.
The daughter, in preparation for building a house on her lot, carefully excavated an area eight feet deep for the purpose of building a basement. The daughter completed construction of her house and macadamized an area for use as a driveway without changing the former contours of the land. Shortly thereafter, the sister began to make complaints to the daughter about the flooding of her basement, which she claimed had been previously free of water.
The sister then built a concrete wall three feet along her border with the daughter to prevent the flow of rain water running onto her land from the daughter’s property. This caused the surface water to stand and become stagnant on the daughter’s land. The daughter demanded that the sister remove the wall, and upon the sister’s refusal, the daughter brought an appropriate action to compel removal.
The most likely result is
(A) the sister must remove the wall because she has no right to obstruct the flow of such surface water.
(B) the sister must remove the wall at the daughter’s expense.
(C) the sister may leave the wall without being liable to the daughter for money damages.
(D) the sister may leave the wall, but she will be liable to the daughter for money damages.
- (C) This Multistate question deals with surface waters. Surface waters are those that come from rain, springs, and melting snow and ice, and simply follow the contours of the land and have not yet reached a natural water course or basin with well- defined bed and banks. Under the common law rule (still followed in a majority of jurisdictions), sometimes called the “common enemy rule” because surface water is considered a common enemy, the lower tract is not burdened with any servitude in favor of the higher land, and the owner of the lower tract has the right to protect his lower tract from “the common enemy” or the flow of surface water by making any improvements that are suitable for the purpose. Choice (A) is incorrect. Under the “common enemy rule” for surface waters, the sister had a right to build the wall to protect her downhill home from the extra water running down into it. Because she does have the right to obstruct the flow of the excess surface water caused by the daughter’s driveway, this choice is incorrect. Choice (B) is incorrect. Under the “common enemy rule” for surface waters, the sister had a right to build the wall to protect her downhill home from the extra water running down into it. Because she does have the right to obstruct the flow of the excess surface water caused by the daughter’s driveway, this choice is incorrect. Choice (D) is incorrect. Not only does the sister have the right to build the wall to protect herself from the excess water caused by the daughter’s new driveway, but the sister would also not be liable to the daughter for the damages caused by the sister’s new wall.
- A landowner, her aunt, and her brother are the owners of three contiguous lots. A downward slope exists from the landowner’s land to the brother’s
land.
The aunt, the owner in the middle of the three lots, was an amateur scientist. She began experimentation with electricity, using a lightning rod to conduct her experiments. Frequently, the electrical storms apparently originated in and over the area within which the brother’s land is located. Because of this, the aunt would often angle her rod over the fence between her property and the brother’s property to maximize her chances of success. After one entire rainy and windy season of experiments, the brother had grown tired of the aunt’s lightning-rod intrusions because they interfered with his ham radio operations in his garage. The brother brought suit to enjoin such lightning-rod experiments. At trial, it was found that the aunt had been conducting her activities in the airspace directly above the brother’s land.
The court should
(A) enjoin the aunt’s experiments because they constitute an unreasonable interference with the space above the brother’s land.
(B) enjoin the aunt’s experiments because one does not have the right to engage in such scientific experiment without permission.
(C) not enjoin the aunt’s lightning rod experiments if they were necessary to protect her property from the electrical storms.
(D) not enjoin the aunt’s experiments because the brother does not own the space above his land.
- (A) The possessor of real property has the right to the exclusive possession of the surface of the ground, the airspace above, and the soil underneath, the extent of which is determined by the exterior boundaries extended vertically upward and downward. Choice (A) is correct because any use of the space above one’s land that is unreasonable, improper, or interferes with the use and enjoyment of the surface can constitute a trespass. Choice (B) is incorrect. The aunt will be enjoined not because she did not have permission; rather, she will be enjoined for the simple reason that her conduct created an interference with the airspace directly above the brother’s home. These Lightning rod experiments directly affected the brother’s ham radio activities and constituted a trespass. Choice (A) more directly addresses the aunt’s conduct and is a better explanation of why she will be enjoined. Choice (C) is incorrect. Even if the experiments were necessary to protect her property from electrical storms, the aunt cannot use this as a defense to her intentionaL trespass into the airspace directly above the brother’s land. If the aunt were that concerned about her property, she could have just as easily angled her lightning rod above her own property. Choice (D) is incorrect. This statement is not true on the facts. The brother does own the airspace directLy above his land and home. Therefore, he is allowed to enjoin the aunt from conducting experiments in the airspace above that interfere with the use and enjoyment of the surface.
- An investor owned a four-story office building located in downtown. The building was old and badly in need of renovation. To finance the improvements, the investor borrowed $125,000 from his friend. As consideration for the loan, the investor executed a promissory note for $125,000 payable to the friend in one year and secured by a mortgage on the building. The mortgage was dated January 1, and was recorded January 2. Thereafter, on February 1, the investor executed a deed absolute on the building and named the friend as grantee. This deed, although absolute in form, was intended only as additional security for the payment of the debt. In order to make judicial foreclosure unnecessary and to eliminate the right to redeem, the investor then delivered the deed to his nephew in escrow with instructions to deliver the deed to the friend if the investor failed to pay his promissory note at maturity.
On January 1 of the following year, the investor failed to pay the note when it came due. Thereupon, the nephew, in accordance with the escrow instructions, delivered the investor’s deed on the office building to the friend, which the friend promptly and properly recorded. Two weeks later, the investor tendered the $125,000 indebtedness to the friend. When the friend refused to accept it, the investor brought an appropriate action to set aside and cancel the deed absolute and to permit the redemption of the building from the friend. Conversely, the friend counterclaimed to quiet title and argued that the deed absolute was intended as an outright conveyance upon default.
The court should enter a judgment that will grant the relief sought by
(A) the investor, but only if the investor can establish that the mortgage takes precedence over the deed absolute because it was executed earlier in time.
(B) the investor, because the deed absolute did not extinguish his right of redemption.
(C) the friend, because the deed absolute effectuated an outright conveyance that extinguished the redemption interest sought to be retained by the investor.
(D) the friend, because the investor is estopped to deny the effect of the deed absolute in conjunction with the escrow arrangement.
- (B) Mortgages is an area on the MBE that is heavily tested. In this particular question, it has long been recognized in equitythata deed absolute intended forsecurity will, in fact, be construed as a mortgage. This is not really surprising when it is remembered that the traditional form of the mortgage was a conveyance subject to defeasance, and that the equity of redemption was created by the equity court to protect the mortgagor after default. In order to preserve this equity of redemption, various rules were formulated to prevent mortgages from limiting (or clogging) the equity of redemption. The most common example of such rules is the principle “once a mortgage always a mortgage.” This, in effect, means that a mortgagee cannot circumscribe the mortgagor’s right to redeem by disguising the transaction as an outright conveyance. In this example, the facts indicate that the investor executed the deed absolute to the friend as additionaL security. Therefore, the deed will not extinguish the investor’s right of redemption because it (the deed absolute) will be construed as a mortgage and not an outright conveyance. Choice (A) is incorrect. This choice falsely suggests that the only way that the investor can prevaiL is by showing that mortgage takes priority because it was executed earlier in time. However, the investor does not need to go to this much trouble just to prove that the deed was not an absolute or outright conveyance. Because the investor executed the deed absolute to the friend only as additional security, there was no intention to make an outright conveyance. The investor prevails because the delivery of the deed was not intended as an absolute or outright conveyance, and the investor does not need to go to the extra effort of showing which was executed first because the deed will be construed as only a mortgage. Choice (C) is incorrect. At common law, the equity of redemption was created by the equity court to protect the mortgagor after default. In orderto preserve this equity of redemption, various rules were formulated to prevent mortgages from limiting (or clogging) the equity of redemption. Because the deed will be treated as a mortgage (and this is especially true given that the facts state that the deed was delivered only for the limited purpose of providing additional security), the right of redemption will not be extinguished. The act of delivering a deed will not be enough to extinguish the right of redemption. Choice (D) is incorrect. The delivery of the deed to the nephew with escrow instructions will still not be enough to overcome the presumption that, at most, this was to be treated as a mortgage with the right of redemption. Never was this arrangement designed to be an outright conveyance. As such, to say that the investor is estopped to deny the actuaL nature of the transaction is to effectively extinguish the right of redemption. Because the right will still exist, the investor will not be estopped to deny the effect of the deed absolute and escrow instructions.
- A man owned a four-story apartment building. The man borrowed $125,000 from his friend to make improvements. As consideration for the loan, the man executed a promissory note for $125,000 payable to the friend in one year and secured by a mortgage on the apartment building. The mortgage was dated January 1, 2008, and was recorded January 2, 2008. Thereafter, on February 1, 2008, the man executed a deed absolute on the apartment building and named the friend as grantee. This deed, although absolute in form, was intended only as additional security for the payment of the debt. In order to make judicial foreclosure unnecessary and to eliminate the right to redeem, the man then delivered the deed to an escrow agent in escrow with instructions to deliver the deed to the friend if the man failed to pay his promissory note at maturity.
On January 1, 2009, the man failed to pay the note when it came due. The next day, the escrow agent delivered the deed to the apartment building to the friend. The friend then properly recorded this deed on January 3. One week later, on January 10, the friend conveyed the apartment building by warranty deed to an investor for the purchase price of $200,000. On January 12, the man tendered the $125,000 balance due to the friend, which he refused to accept. The man now brings an appropriate action against the friend and the investor to set aside the conveyance and to permit the redemption of the property by the man.
Which of the following best states the man’s legal rights, if any, in his action against the friend and the investor?
(A) The man has no rights against the investor, but the man does have an action for redemption against the friend for the value of the property.
(B) The man has no rights against the friend, but the man does have an action for redemption against the investor for the value of the property.
(C) The man has the option of seeking redemption against either the friend or the investor for the value of the property, but the man cannot set aside the conveyance.
(D) The man has no rights against either the friend or the investor because he defaulted on the promissory note.
- (A) When the “mortgagee” under a deed absolute mortgage transfers to a bona fide purchaser, the mortgagorhas no rights against the bona fide purchaser, but he does have an action for redemption against the “mortgagee for the value of the land, or, at his election, the proceeds of the sale.” The theory is that the mortgagee now has the value of the Land in his hands as a separate fund, and such fund (as a substitute for the land) may be redeemed by the mortgagor. Applying this rule to our given set of facts, the man has no right against the investor, the bona fide purchaser, but he does have an action for redemption against the friend, the mortgagee. Choice (B) is incorrect. When the “mortgagee” under a deed absolute mortgage transfers to a bona fide purchaser, the mortgagor has no rights against the bona fide purchaser, but he does have an action for redemption against the “mortgagee for the value of the land, or, at his election, the proceeds of the sale.” The theory is that the mortgagee now has the value of the land in his hands as a separate fund, and such fund (as a substitute for the land) may be redeemed by the mortgagor. Applying this rule to our given set of facts, the man has no right against the investor, the bona fide purchaser, but he does have an action for redemption against the friend, the mortgagee. Because this choice states that the man has no rights against the friend for the proceeds that the friend is holding, it is incorrect. Choice (C) is incorrect. When the “mortgagee” under a deed absolute mortgage transfers to a bona flde purchaser, the mortgagor has no rights against the bona fide purchaser, but he does have an action for redemption against the “mortgagee for the value of the land, or, at his election, the proceeds of the sale.”The theory is that the mortgagee now has the value of the Land in his hands as a separate fund, and such fund (as a substitute forthe land) maybe redeemed by the mortgagor. Applying this rule to our given set of facts, the man has no right against the investor, the bona fide purchaser, but he does have an action for redemption against the friend, the mortgagee. However, the man does not have an election of rights with regard to the right of redemption; the man may only seek redemption against the friend and not the investor, the bona fide purchaser. Choice (D) is incorrect. Even though the man may have defaulted on the promissory note, he will still have an equitable right of redemption to reclaim the property or, in this case, the proceeds of the sale. However, the man may only seek redemption against the friend and not the investor, who took as a bona fide purchaser under the facts.
- A millionaire owned two adjacent 10-story commercial buildings. One building housed medical offices, and the other building housed dental offices. The first floors of both buildings were occupied by various retail establishments. The buildings’ other floors were rented to professionals and used as offices. There was an enclosed walkway that connected the second floor of each building. Thus, shoppers and office staff could walk across the common walkway and gain access to each building.
While the buildings were being used in this manner, the millionaire sold the dental building to an investor by warranty deed, which made no mention of any rights concerning the walkway. The walkway continued to be used by the occupants of both buildings. Thereafter, the walkway became unsafe as a consequence of wear and tear.
As a result, the investor hired a contractor to repair the walkway area. When the millionaire saw the contractor removing the carpeting along the walkway, he demanded that the investor discontinue the repair work. After the investor refused, the millionaire brought an action to enjoin the investor from continuing the work.
The most likely result will be a decision for
(A) the millionaire, because the investor does not have rights in the walkway.
(B) the millionaire, because the investor’s rights in the dental building do not extend to the walkway.
(C) the investor, because the investor has an easement in the walkway and an implied right to keep the walkway in repair.
(D) the investor, because he has a right to take whatever action is necessary to protect himself from possible tort liability from persons using the walkway.
- (C) The investor would have an implied easement in the walkway. An implied easement is created and proved, not by the words of the conveyance, but by all the circumstances surrounding the execution of the conveyance. It is based on the intention of the parties as inferred from the surrounding circumstances. There are five distinct requirements for the existence of an implied easement, all of which are present in the facts of this example. First, there must be two properties owned by one person who uses one of the pieces of property to serve the other piece of Land. Second, there must be a conveyance of one part of the property to another person, the other part being retained by the conveyor. Third, the quasi-easement must be apparent at the time of the conveyance. Fourth, the quasi-easement must be continuous, which means that the use of the quasi-servient tenement must be permanently adapted to serve the needs of the quasi-dominant tenement. Fifth, the quasi-easement must be (a) “reasonably necessary” to the convenient enjoyment of the quasi-dominant land if that tract is the property conveyed to the grantee, and (b) “strictly necessary” to the enjoyment of the quasi-dominant tenement if that tract is retained by the grantor. By virtue of the implied easement, the investor has the right to enter the walkway for the purpose of repairing, maintaining, and improving the means by which the easement is enjoyed. Choice (A) is incorrect. The investor has an implied easement in the walkway. Since all of the five requirements of an implied easement are present in this question, the investor does have rights in the walkway. One of those rights is the right to enter the walkway to maintain and improve it. Choice (B) is incorrect. One of the rights that the investor does have is the right to use the walkway that connects his building to the other building. Because use of the walkway is reasonably necessary to the convenient enjoyment of the investor’s building, it is one of the investor’s rights. Choice (D) is incorrect. This choice suggests that the investor has unfettered discretion to do whatever is necessary to protect himself from tort liability. While that is something that the investor would want to be concerned with, he is not in a position to take any action he deems necessary. Because both the investor and the millionaire share the walkway, the investor cannot do anything to the walkway that would encroach upon the millionaire’s rights in this walkway because both parties are allowed reasonable use and enjoyment of the walkway.
- A grantor conveyed his property to his son “for life, remainder after (the son’s) death to his heirs.” Two years later, the son entered into a real estate agreement for the sale of the property to his friend, whereby the son agreed to convey the premises to his friend in fee simple absolute. Prior to the settlement date, the friend contacted the son, telling him that he would not perform his part of the agreement because the son could not convey a fee simple.
If the Rule in Shelley’s Case is followed in this jurisdiction, the outcome of a suit by the son for specific performance of the real estate contract would result in
(A) the son’s not succeeding, because he could not convey marketable title.
(B) the son’s succeeding, because he had a fee simple to convey.
(C) the son’s not succeeding, because his heirs have to join in the transaction in order to convey marketable title.
(D) the son’s succeeding, because the son’s conveyance of his life estate to the friend divested the contingent remainder of his heirs.
- (B) The son would succeed because the Rule in Shelley’s Case operates as follows: If a life estate is conveyed to Ann and in the same instrument, a remainder is given to Ann’s heirs, then Ann will take a remainder in fee simple. In other words, Ann’s life estate merges with the remainder to her heirs, thus giving Ann a fee simple absolute. Choice (A) is incorrect. After application of the Rule in Shelley’s Case, the son will take a fee simple absolute, which would allow him to convey it to his friend. This would give the son marketable title to the property. Choice (C) is incorrect. After application of the Rule in Shelley’s Case, the son will take a fee simple absolute. Because the son has outright fee ownership, his heirs do not need to be joined in the transaction in order for the son to be able to convey marketable title. The son has marketable title all on his own because he takes all of the property in fee simple absolute. Choice (D) is incorrect. The son will succeed, but not for the reasons given in this choice. This choice suggests that the son’s conveyance of his own life estate to the friend was the reason that his heirs were divested of their remainder. The remainder was divested by application of the Rule in Shelley’s Case because it is followed in this jurisdiction, not from anything the son himself did. Also, because the Rule in Shelley’s Case operated to give the son a fee simple absolute, he was not conveying just a life estate to the friend; the son was conveying a fee simple interest.
- A grantor conveyed her mansion and surrounding property to her nephew “for life, remainder after (the nephew’s) death to his heirs.” Two years later, the nephew entered into a real estate agreement for the sale of the property to his brother, whereby the nephew agreed to convey the premises to the brother in fee simple absolute. Prior to the settlement date, the brother contacted the nephew, telling him that he would not perform his part of the agreement because the nephew could not convey a fee simple.
The Rule in Shelley’s Case has been abolished by statute in this jurisdiction. Thus, the nephew’s prayer for specific performance would be
(A) denied, because the Rule would not be triggered, thus creating only a life estate in the nephew.
(B) granted, because the remainder in his heirs would become vested into a full fee in those heirs.
(C) granted, because the nephew’s heirs receive a vested indefeasible interest in the property.
(D) denied, because under the Doctrine of Worthier Title, at the termination of the nephew’s life estate, the grantor has a reversionary interest.
- (A) This choice is correct because since the Rule was abolished, the nephew would acquire only a life estate. As a result, in order to convey marketable title, the nephew’s heirs must join in the conveyance of the property. The remainder to the nephew’s heirs is treated as a contingent remainder that does not merge with the nephew’s life estate. Choice (B) is incorrect. This choice suggests that the remainder in the nephew’s heirs would become vested into a full fee in those heirs. However, that would not occur until the nephew’s death. Because the nephew is still alive on these facts, the heirs do not own a full fee. Choice (C) is incorrect. The nephew’s heirs received a contingent remainder in the property. A contingent remainder is any remainder created in favor of an existing but unascertained person. Because it is not clear who the nephew’s heirs will be upon his death, the “heirs” are unascertamed and the remainder is contingent, not vested. Moreover, this choice is incorrect because the nephew’s prayer for specific performance will not be granted; the nephew will not be able to compel the brotherto purchase the property because the nephew did not have a fee simple to grant. Choice (D) is incorrect. The Doctrine of Worthier Title (or the Rule against Remainders in the grantor’s heirs) is not at issue here. That rule arises in an example such as this: Al conveys to Bea for life, remainder to Al’s heirs. Under the Doctrine of Worthier Title, Bea would still have her life estate, and the remainder to Al’s heirs would become a reversion in Al himself.
However, in this question, the remainder is going to the heirs of the life tenant, not the grantor. So, the Doctrine of Worthier Title is inapplicable here.
- A homeowner conveyed his property to his cousin “for life, with remainder to (the homeowner’s) heirs.” Later, the cousin conveyed the property to a farmer “for as long as he would continue to farm the property.” After the cousin’s death, the homeowner’s heirs brought suit against the farmer to quiet title. Which common law doctrine would be most applicable in the court’s determination of the ownership of the property?
(A) Rule in Wild’s Case.
(B) Doctrine of Destructibility of Contingent Remainders.
(C) Doctrine of Worthier Title.
(D) Rule against Remainders in the Grantees’ Heirs.
- (C) Choice (C) is correct because the old common law Doctrine of WorthierTitle is construed today as a rule of construction whereby the grantor presumes not to create a remainder in his heirs, but rather intends to retain a reversion in himself. Refer to Justice Cardozo’s opinion in the leading case of Doctorv. Hughes, 22 N.E. 211,1919. Choice (A) is incorrect. The Rule in Wild’s Case, when applied to a devise such as “to B and his children” will be construed to mean a life estate to B and a remainder to B’s children if B has no living children at the time of the devise. If B had children at the time of the devise, B and B’s children would be tenants in common. Note that the rule only applies to devises. In this question, the homeowner conveyed to the cousin so the Rule is inapplicable on these facts. Choice (B) is incorrect. This common law doctrine was applicable when a life estate had terminated and the holder of a contingent remainder had not met the condition, for whatever reason, If the remainder holder did not fulfill the condition by the time the life tenant died, the remainder was destroyed and the estate returned to the grantor permanently, even if that person eventually fulfilled the condition. The destructibility rule has been abolished in most but not all states. However, that is inapplicable here because the life tenant has not died, and reversion back to the grantor is not in issue here. Choice (D) is incorrect. The Rule against Remainders in the grantees’ heirs is more commonly known as the Rule in Shelley’s case. The Rule in Shelley’s Case operates as follows: If a life estate is conveyed to A and in the same instrument, a remainder is given to A’s heirs, then A will take a remainder in fee simple. In other words, A’s life estate merges with the remainder to his heirs, thus giving A a fee simple absolute. However, that Rule is inapplicable here because the original grant was “to (the cousin) for life, remainder to (the homeowner’s) heirs.” Because the remainder was given to the grantor’s heirs and not the life tenant’s heirs, the Rule in Shelley’s Case is not applicable.
- A farmer owned a 40-acre tract of farmland located in a small southern town. The farmer leased the property and building thereon to a tenant for a term of seven years commencing on February 15, 2000 and terminating at 12:00 noon on February 15, 2007. The lease contained the following provision:
“Lessee covenants to pay the rent of $5,000 per month on the 15th day of each month and to keep the building situated upon said leased premises in as good repair as it was at the time of said lease until the expiration thereof.” The lease also contained a provision giving the tenant the option to purchase 10 acres of the tract for $150,000 at the expiration of the lease term. Before the lease was executed, the farmer orally promised the tenant that he (the farmer) would have the 10-acre tract surveyed.
During the last year of the lease, the tenant decided to exercise the option to purchase the 10 acres of the tract. Without the farmer’s knowledge, the tenant began to build an irrigation ditch across the northern section of the property. When the tenant notified the farmer that he planned to exercise the option, the farmer refused to perform. The farmer also informed the tenant that he never had the 10-acre tract surveyed.
If the tenant brings suit for specific performance, which of the following is the farmer’s best defense?
(A) The option agreement was unenforceable under the parol evidence rule.
(B) The farmer’s failure to survey the 10-acre tract excused him from further obligations under the contract.
(C) The description of the property was too indefinite to permit the remedy sought.
(D) The option was unenforceable because it lacked separate consideration.
- (C) Smith and Boyer note that no conveyance is valid unless the description of the land sought to be conveyed is sufficient to identify the land. The facts indicate that the tract is a 40-acre tract of farmland. The leasehold agreement provided that the tenant would have an option to purchase 10 acres of the tract. Because the lease failed to identify or describe a distinct piece of the tract, the farmer’s best argument is that the option should fail for lack of description. Law of Property, pg. 300. Choice (A) is incorrect. Under the parol evidence rule, evidence of a prior or contemporaneous agreement is inadmissible if it would vary or contradict the terms of a totally integrated writing. There is no need to consider extrinsic evidence here because the original lease agreement already contained a provision giving the tenant the option to purchase 10 acres of the tract. As a result, the parol evidence rule is inapplicable. Choice (B) is incorrect. The better defense here is that the description is so insufficient as to render the prior agreement unenforceable. It is no defense for the farmer to argue that something he himself orally promised (to have the tract surveyed) and then failed to do can excuse his performance under the contract. Because this choice fails to take into account the best reason why the lease fails to satisfy the statute of frauds (inadequate description), it is not correct. Choice (D) is incorrect. Even if the option to purchase had been fully supported by its own consideration, it still would have failed due to its lack of specificity. Choice (0) would be a poor defense because it fails to address the strongest reason why the suit for specific performance will fail—that the description was so insufficient as to render the agreement unenforceable.
- A landlord was the owner of a large, high-rise apartment building in a Midwestern city. On June 1, 2007, two tenants took possession of a three- bedroom apartment in the landlord’s building under a three-year lease at a rental of $1,200 per month. Their lease (as all other leases given by the landlord) contained the following provisions:
“The term of this lease shall be three years from the date hereof as long as all the agreements herein shall be faithfully performed.”
The two tenants lived in the apartment for two years. On June 10, 2009, however, a fire destroyed the apartment building. As a result, all the apartments in the building were rendered uninhabitable. After the two tenants were dispossessed from their apartment, the landlord brought suit against them to recover the rent due for the balance of the lease. The two tenants claim that they are no longer liable for rent or any other obligations under the lease. The landlord— tenants leasehold contract contained no provision regarding liability for fire.
If the decision is in favor of the two tenants, it will most likely be because
(A) there was nothing in the lease regarding liability for fire.
(B) the two tenants did not own an interest in the property.
(C) the jurisdiction has rejected the common law view on the tenant’s duty to pay rent.
(D) the landlord did not contract to convey the property to the two tenants.
- (C) At common law, a tenant remains liable to pay rent even though because of fire, floods, storms, or other action of the eLements or otherwise, the property is rendered totally uninhabitable unless the lease otherwise provides. So, if the decision is in favor of the two tenants, it will most Likely be because the strict common law view has been rejected by this jurisdiction and another view has been adopted regarding tenant liability for rent. Though it is not clear that a court would come to this conclusion, the call of the question requires you to presume that the tenants would win and to find the best reason why the court could come to this conclusion. Choice (A) is incorrect. At common law, a tenant remains liable to pay rent even though because of fire, floods, storms, or other action of the elements or otherwise, the property is rendered totally uninhabitable unLess the lease otherwise provides. So, a lease provision allocating risk would be the most helpful to the tenants. However, the lease’s silence on liability does not automatically help the tenants either because if the jurisdiction still retained the common law view, the tenants would still be liable for rent. Because choice (C) explicitly states that the jurisdiction has rejected the common law view, it provides a stronger reason than choice (A). Choice (B) is incorrect. First, this choice is incorrect because a leasehold is considered an interest in land. Moreover, because the common law duty to pay rent was absolute, if the jurisdiction retained this rule, this argument still doesn’t explain why the tenants should be excused from paying their rent. Choice (D) is incorrect. The doctrine of equitable conversion is inapplicable. Note that the equitable conversion doctrine applies only when there is an enforceable obligation to sell land. So, even though this is a true statement on the facts, this argument still doesn’t explain why the tenants should be excused from paying their rent.
- A homeowner executed a deed by which he conveyed his home and surround property for a consideration of one dollar, receipt of which was acknowledged, “to my daughter for life, then to my aunt for life, but if my aunt moves to another state, to my sister for the life of my aunt, then to the heirs of my aunt if my aunt does not move to another state, and to the heirs of my sister if my aunt does move to another state.” This deed was promptly recorded.
During the daughter’s lifetime, the aunt’s interest may best be described as a
(A) contingent remainder.
(B) shifting executory interest.
(C) vested remainder subject to complete divestiture.
(D) vested remainder subject to partial divestiture.
- (C) The aunt’s interest in the property would be a vested interest subject to complete divestiture. At the expiration of the daughter’s life estate, the aunt would immediately be entitled to take possession of the property. However, the aunt’s vested remainder (i.e., life estate in futuro in the property) would be subject to complete divestiture upon the contingency of her redomiciling. Although it may seem Like the aunt is only getting a life estate because of the language “then to my aunt for life,” the aunt is receiving a vested remainder because if she never moves, it goes to the aunt (stays with the aunt) and then passes to the aunt’s heirs. Choice (A) is incorrect. A contingent remainder is a remainder that is limited because it will depend on an event or condition that may not happen or may not be performed until after the termination of the preceding estate. ConverseLy, we are certain here that the daughter’s life estate will eventually terminate. Thus, although the aunt may predecease the daughter, thereby terminating the aunt’s own actual enjoyment, the aunt’s right to such enjoyment (i.e., vested remainder) is not uncertain. Because the property is designed to pass to the aunt (as long as the aunt abides by the one condition) the remainder is not contingent. Choice (B) is incorrect. A shifting executory interest “cuts short” or terminates a preceding estate in favor of another grantee, and it shifts the right of possession from one grantee to another upon the happening (or non-happening) of a particular contingency. In order for the aunt to take, she must wait for the daughter to pass away. There is no contingency that can allow the aunt to take away the daughter’s Life estate any earlier than the daughter’s death. Because of this lack of a contingency, the aunt’s future interest is a remainder, not an executory interest. Choice (D) is incorrect. A remainder is vested subject to being partly divested when the remainderman is in existence and ascertained, but the amount of her estate is subject to diminution in favor of other members of a class. This type of remainder, frequently called a remainder vested subject to open, is often illustrated by a class gift. Because the aunt is not part of a class that could dilute the aunt’s property interest, this is not the name of the estate the aunt was given.
- A grantor executed a deed by which he conveyed his apartment building for a consideration of one dollar, receipt of which was acknowledged, “to my son for life, then to my uncle for life, but if my uncle moves to another state, to my brother for the life of my uncle, then to the heirs of my uncle if my uncle does not move to another state, and to the heirs of my brother if my uncle does move to another state.” This deed was promptly recorded.
During the son’s lifetime, the brother’s interest may best be described as a (an)
(A) estate pur autre vie.
(B) contingent remainder pur autre vie.
(C) vested remainder pur autre vie.
(D) shifting executory interest pur autre vie.
- (D) The brother’s interest would be a shifting executory interest purautre vie. An executory interest is an interest that divests the interest of another transferee (shifting executory interest) or that “follows a gap” or divests the interest of the transferor (springing executory interest). Because the brother’s interest could divest the uncle of the uncle’s interest in the property (if the uncle redomiciles), the interest is an executory interest of a shifting type. Another way to put this is that the brother can only take if the uncle meets (or fails to meet) a particular contingency, and that categorizes the brother’s future interest as an executory interest. Furthermore, his shifting executory interest would be pur autre vie, because it would be an estate for the life of another (the uncle). Choice (A) is incorrect. This choice is incomplete. All it states is that the brother received an estate pur autre vie. An estate pur autre vie means that the duration of the estate is measured by the life of someone other than the person receiving it. However, this choice still doesn’t tell us what type of estate or future interest was given to the grantee (i.e., life estate, shifting executory interest). Because this choice only states the duration without explaining the type of interest granted, it is incorrect. Choice (B) is incorrect. A remainder is a future interest created in a third person that is intended to take after the natural termination of a preceding estate. A contingent remainder is any remainder that is created in favor of an ascertained person but (a) is subject to a condition precedent, or (b) is created in favor of an unborn person, or (c) is created in favor of an existing but unascertained person. Choice (B) is incorrect because the brother’s interest in the property will not come after the natural termination of a preceding estate (like a life estate). Rather, the brother will have to wait to see if the uncle redomkiles in order for the brother to take the property. Also, when this contingency occurs, it will divest another grantee of the estate. Because the brother will not take after the natural termination of a preceding estate, his interest cannot be categorized as a remainder. Choice (C) is incorrect. A remainder is a future interest created in a third person that is intended to take after the natural termination of a preceding estate. A vested remainder is a remainder created in an ascertained and existing person that is not subject to any condition precedent except the normal termination of the preceding estate. Choice (C) is inco rrect because the brother’s interest in the property will not come after the natural termination of a preceding estate (like a life estate). Rather, the brother will have to wait to see if the uncle redomiciles in order for the brother to take the property. Also, when this contingency occurs, it will divest another grantee of the estate, Because the brother will not take after the natural termination of a preceding estate, his interest cannot be categorized as a remainder.
- In 1998, a landowner owned a 30-acre tract located just inside the city. The tract included the family home, a decaying antebellum mansion complete with tennis courts, stables and a smaller second house that was once occupied by tenants who farmed the city. The second house, however, had long been vacant as a result of the economic decay of the surrounding area.
Prosperity burst upon the city in 1999, and the landowner began selling acre lots in the tract. By 2006, the landowner had sold 25 acres, retaining five acres that included the antebellum mansion, tennis courts, stables, and the former tenants’ house.
On May, 19, 2007, the landowner entered into a valid written contract with a buyer. According to the terms of their agreement, the landowner agreed to sell and convey his remaining interest in the tract for a consideration of $500,000. The land sale contract provided a closing date of November 19, 2007 and stipulated that “time was of the essence.”
On July 2, 2007, a fire destroyed the antebellum mansion. The landowner had the mansion insured for $450,000 against fire loss and collected that amount from the insurance company. At the closing on November 19, the buyer tendered a cashier’s check for $50,000 and demanded a deed conveying a fee simple interest in the property. Conversely, the landowner tendered a deed of conveyance and demanded the full purchase price of $500,000. The buyer refused the landowner’s demand.
In an appropriate action for specific performance against the buyer, the landowner demanded $500,000. If the landowner prevails, which of the following is the best rationale for the outcome?
(A) The fact that the antebellum mansion was insured for $450,000 is irrelevant.
(B) The landowner and the buyer each had an insurable interest in the property.
(C) The doctrine of equitable conversion has been abolished.
(D) The doctrine of equitable conversion requires such a result.
- (D) This is exactly the same situation that Smith and Boyer discuss in their hornbook. An executory contract for the sale of land requiring the seller to execute a deed conveying legal title upon payment of the full purchase price works an equitable conversion in order to make the purchaser the equitable owner of the land and the seller the equitable owner of the purchase price. The result is that the purchaser, as equitable owner of the land, takes the benefit of all subsequent increases in value and, at the same time, becomes subject to all losses not occasioned by the fault of the seller. Thus, the purchaser, to protect himself, either must procure his own insurance, or by appropriate provision in the contract, cast the risk upon the seller. He is not, however, entitled to recover insurance payments payable to the vendor. Choice (D) is a better answer than (A) because the equitable conversion doctrine (which places the risk on the buyer) requires such a result. Choice (A) is incorrect. Though it could be argued that the court awarded the landowner the $500,000 because the court chose to ignore the insurance policy, this choice still does not explain why this particular result was required. The question of why the court still granted the landowner his $500,000 still remains. It is because the court followed the common Law default doctrine of equitable conversion that the landowner was stiLL awarded the purchase price. Though choice (A) may be factually correct and the court may have chosen to ignore the insurance policy, choice (D) is preferred because it states the correct rule of law that the court followed to reach this result. Students wiLl want to choose a correct statement of law over one of fact because a correct rule of law is more universal and can apply to many factual situations. A correct statement of fact, however, may only be applicable to one factual scenario. Choice (B) is incorrect. Even if both the landowner and the buyer each had an insurable interest in the property, this still does not explain why the landowner was awarded the fulL $500,000 by the court. It is because the court followed the common law default doctrine of equitable conversion that the Landowner was still awarded the purchase price. Though (B) may be factuaLLy correct and both parties had an insurable interest in the property, choice (D) is preferred because it states the correct rule of law that the court foLlowed to reach this result. Choice (C) is incorrect. This is the exact opposite of the correct answer and would be a reason NOT to award the landowner the $500,000.
- On November 1, Beeson contracted to purchase from Sloan for $250,000 certain property located in the City of La Mirada. In the contract of sale, Beeson and Sloan agreed that the property was to be used for the purpose of building a commercial shopping mall. The contract required Beeson to pay Sloan a deposit of $12,500, with the balance of the purchase price payable at closing a month later on December 1. On November 24, the city council rezoned the property so that it could be used only for single-family residential purposes.
As a consequence, Beeson refused to honor the contract. Sloan now brings an action for specific performance arguing that the doctrine of equitable conversion places the loss on the buyer. Beeson argues that to enforce the contract would be harsh and oppressive to him.
If judgment is for Beeson, it will most likely be because
(A) Sloan assumed the risk.
(B) Sloan would be unjustly enriched.
(C) legal title remained in Sloan.
(D) equity will relieve Beeson of a bad bargain.
- (B) Under the doctrine of equitable conversion, the risk of loss from casualty and other fortuitous events is normally placed on the purchaser in the absence of controlling provisions in the contract. Equity thus considers the vendee as the owner of the land and the vendor as the owner of the purchase money. Smith and Boyer point out, however, that this rule is limited in its application to cases where the intention of the parties will not produce an inequitable result. For exampLe, assume that A contracts to seLl to B a certain piece of land that was to be used for the purpose of erecting a hotel. However, between the time the contract of sale was made and the time for delivery of the deed, the city council rezones the Lot so that it could only be used for residential purposes. A now brings suit for specific performance. In this situation, Smith and Boyer note that the granting of specific performance would be unduly harsh and oppressive to B. Because the intent of the parties was defeated by the supervening event, specific performance should be denied. However, the denial of specific performance does not end the matter, but, instead, the vendor may proceed against the purchaser in a suit at Law for any damages that have been incurred, but the purchaser does not have to be required to tender the purchase price of the property. By analogy, if the court rules in favor of Beeson, it will be to avoid unjust enrichment. Choice (A) is incorrect. Under the doctrine of equitable conversion, the risk of loss is normally pLaced on the buyer of the property, absent any controlling provisions in the contract. Here, Sloan (as the seller) did not assume the risk of the supervening event that the city council would rezone the property. If anyone assumed the risk of any unforeseen events, it would have been Beeson (as the buyer). Choice (C) is incorrect. Under the doctrine of equitable conversion, legal title to the property remains with the seller, and the buyer is treated as the equitable owner of the land. Given that this is the traditional definition, this is the reason why Beeson should be required to buy the property. Stating that Sloan retains legal title only enforces the idea that Beeson should be forced to retain a bad bargain. Because this choice fails to explain why Beeson should prevail, it is incorrect. Choice CD) is incorrect. This choice falsely suggests that the court will inquire into the imprudence of the bargain struck between the two parties. It is not for the court to decide whether this was a good bargain or one formed from bad judgment. However, the court (to do equity) may avoid unjust enrichment by refusing to grant specific performance to the seller and rescind the contract of sale. However, it would be due to the supervening illegality of the zoning ordinance, not simply because itwas a bad bargain looking backwards.
- A state has the following recording statute in effect:
“No conveyance is good against a subsequent purchaser for a valuable consideration and without notice, unless the same be recorded prior to subsequent purchase.”
An owner is the fee simple the owner of a 20-acre tract of unimproved land, situated in the state. On May 1, the owner sold the tract to a buyer for the purchase price of $40,000 under a quitclaim deed. The owner delivered the deed to the buyer who did not record the deed. After the sale to the buyer, the owner found himself in desperate need of money because he lost his job. So the owner, in consideration of the sum of $75,000, conveyed the tract to an investor by warranty deed. This transaction took place on August 1. When the investor acquired title to the tract, he had no actual knowledge of the buyer’s deed (which was still unrecorded). On August 10, the buyer recorded his deed to the tract. The investor did not record the deed he received from the owner until August 15.
In an appropriate action to quiet title to the tract, in which all interested parties have been joined, title will be found to be in
(A) the buyer, because his deed preceded the investor’s deed.
(B) the buyer, because his deed was recorded prior to the investor’s deed.
(C) the investor, because he is protected by the recording statute.
(D) the investor, because he took title by warranty deed and the buyer took title by quitclaim deed.
- (C) The keyto this question is carefully interpretingthe wording of the recording statute. According to the statutory Language, the state’s recording act protects subsequent bona fide purchasers for value and without notice. As such, this is an example of a pure “notice” type recording statute, which generally provides that an unrecorded conveyance or other instrument is invalid as against a subsequent bona fide purchaser forvalue and without notice. Under this type of recording statute, the subsequent bona fide purchaser prevails over the prior interest whether the subsequent purchaser records ornot. As a consequence, the investor will prevail over the buyer because the investor was a subsequent bona fide purchaser without notice of the buyer’s deed. Choice (A) is incorrect. Even though the buyer’s deed preceded the investor’s deed, that fact will not be enough for the buyer to prevail. Because the state has a notice recording statute, a subsequent purchaser for valuable consideration and without notice can still prevail, even though someone else received a prior deed. Choice (B) is incorrect. While this is true, the test isn’t who recorded first; rather, the test is whether the subsequent purchaser for valuable consideration purchased without notice. Because the buyer’s deed remained unrecorded when the investor purchased the property and because the investor had no actual notice of the grant to the buyer, the investor will prevail in a notice jurisdiction, even though the buyer ultimately recorded before the investor. Choice (D) is incorrect. A quitclaim deed is a deed that makes none of the six covenants of title. Whether a grantee took title via a quitclaim or warranty deed is only relevant when a grantee is suing a grantor under a deed covenant. Because the competition here is between two grantees, the only relevant inquiry is whether or not the investor had notice when he purchased from the owner. The fact that the investor has covenants in his deed will not help him if he had either actual or constructive notice of the grant to the buyer. Because this choice falsely suggests that deed covenants would be the reason why the investor prevails over the buyer, it is in correct.
- A deed executed by a grantor conveyed his property for a consideration of one dollar, receipt of which was acknowledged, “to my brother for life, then to the heirs of my brother.” A life interest in the property for the life of the brother is worth $20,000 on the date of the conveyance. The total worth of the property is $50,000. The brother accepted but didn’t record the deed. The recording statute in this jurisdiction provided “unless recorded, all written instruments affecting title to land are void as to subsequent purchasers who paid value and without notice.”
Four years later, the grantor purported to convey his property in fee simple absolute to his two sons, by a warranty deed, as a gift. The two sons recorded the deed. Shortly thereafter, the brother ascertained that the grantor’s sons were about to take possession of the property. As a consequence, the brother promptly recorded his deed.
In a dispute between the brother and the grantor’s children as to the ownership of the property, if the brother prevails it will be because
(A) the brother paid valuable consideration for his deed.
(B) the brother recorded his deed before the grantor’s children sought to oust him from the land.
(C) the grantor’s sons are not protected by the recording statute.
(D) the grantor’s knowledge is imputed to his children.
- (C) In order to be a bona fide purchaser protected under the recording act, one must (a) be subsequent; (b) pay value; (c) be without notice (the value must have actually been paid before notice); and (d) be of good faith. Recording statutes do not protect a subsequent claimant who has not paid more than a nominal consideration because he is nota purchaser. Therefore, in our case, the grantor’s children are not protected by the recording statute because they are not purchasers (as the grantor purportedly conveyed the property to them as a gift). The brother will ultimately prevail because he was the first in time to take the property from the grantor. Choice (A) is incorrect. In order to be a bona fide purchaser protected under the recording act, one must (a) be subsequent; (b) pay value; (c) be without notice (the value must have actually been paid before notice); and (d) be of good faith. In this type of situation, part of the inquiry would be whether or not the subsequent grantees paid valuable consideration because if a subsequent grantee paid valuable consideration and took without notice, then that person would prevail even though someone else had already been granted the property. However, this type of inquiry only matters to subsequent grantees. Because the brother was the first grantee, the manner in which the brother took the property is irrelevant. It makes no difference whether the brother paid or it was given to him as a gift, because if the subsequent grantees took without nottce and paid valuable consideration, the brother would lose even if he paid. Because this answer choice falsely suggests that the brother’s payment of consideration is the reason he prevails, it is incorrect. Choice (B) is incorrect. An ouster occurs where one co-tenant manages to wrongfully exclude her co-tenants from possession of the property. When one co-tenant ousts her cotenant from possession, the ousted tenant has a cause of action against the possessor to regain possession for herself with the possessor. In this fact pattern, there is no co-tenancy relationship between the brother and the grantor’s children where ouster would be a viable issue to raise. The brother prevails because the sons cannot claim protection under the recording statute, not because the brother recorded before there was any ouster. Choice (D) is incorrect. If the sons had actual knowledge of the transaction to the brother or if there were record notice (assuming the brother did record first), then the children would possess their own knowledge of the prior transaction. However, the grantor’s knowledge of the prior transaction to the brother is not necessarily imputed to the grantor’s children simply because they are the grantor’s children; the children have to know about it in their own right.
- A father died leaving a will by which he devised a 100-acre tract to his daughter. At the time of the father’s death, he believed that he owned all of the 100-acre tract. However, the father actually owned 95 acres of the tract. The other five acres were owned by a farmer. After taking possession of the tract, the daughter executed and delivered a warranty deed to a buyer, purporting to convey all 100 acres of the tract. The agreed purchase price was $100,000. According to the terms of the sale, the buyer paid the daughter a down payment of $25,000, with the unpaid portion of the purchase price (i.e., $75,000) secured by a mortgage. The mortgage instrument described the property interest as covering all 100 acres of the tract.
After the daughter—buyer transaction was completed, the farmer came forward and informed the parties that she was the true record owner of five acres of the 100-acre tract. Upon the farmer’s threat to sue to quiet title, the daughter negotiated a deal whereby she paid the farmer $5,000 for the five-acre tract. As part of their agreement, the farmer executed and delivered a quitclaim deed quitclaiming to the buyer all of her interest in the five-acre parcel.
Thereafter, the buyer defaulted on the mortgage debt, and the daughter properly consummated foreclosure proceedings, becoming the purchaser of the tract at her own foreclosure sale. The description of the deed in the foreclosure sale referred to the tract as consisting of all 100 acres. After the foreclosure sale was finalized, the buyer brought suit claiming title in fee simple to the five-acre tract, formerly owned by the farmer.
In an appropriate action to determine the title to the said five-acre tract, the buyer will
(A) lose, because the daughter did not have good and marketable title at the time she purported to convey the 100-acre tract to the buyer.
(B) lose, because the doctrine of after-acquired title controls.
(C) win, because the deed from the farmer to the buyer was a quitclaim deed.
(D) win, because the quitclaim deed from the farmer to the buyer was subsequent to the deed from the daughter to the buyer and to the mortgage.
- (B) Under the doctrine of estoppel by deed (sometimes referred to as the “after-acquired title” doctrine), when a person executes a deed purporting to convey an estate in Land that he does not have (orwhich is largerthan he has), and such person at a later date acquires such estate in such land, then the subsequently acquired estate will, by estoppel, pass to the grantee. In this example, once the daughter purchased the five-acre tract from the farmer, title inured to the benefit of the buyer as evidenced by the farmer’s quitclaim deed to the buyer. Most importantly, the daughter held a mortgage on the 100-acre tract. The mortgage instrument described the property interest as covering all 100 acres of the tract. Thus, when the buyer defaulted and the daughter foreclosed, she purchased back the entire tract of land (including the five acres previously owned by the farmer). Choice (A) is incorrect. This choice is wrong because even though the daughter did not have good and marketable title to all of the tract initially, this title defect was subsequently cured by operation of the daughter—farmer transaction. Choice (C) is incorrect. At the moment the daughter purchased and acquired title to the five acres from the farmer, title automatically inured to the benefit of the buyer by virtue of the estoppel-by-deed doctrine. It is irrelevant whether the farmer quitclalmed the property to the buyer. This is tricky because in most states estoppel by deed will not be applied where the conveyance is by a quitclaim deed. In this question, however, the daughter—buyer conveyance was by warranty deed. For this reason, the doctrine of estoppel by deed will apply. In other words, title would nonetheless inure to the buyer’s benefit regardless of the form of deed used. Choice (D) is incorrect. All this choice does is mention some of the facts that occurred subsequent to the deed from the daughter to the buyer and the mortgage. Certainly, the quitcLaim deed from the farmer to the buyer came after the daughter—buyer deed and the mortgage. However, this choice does not explain why those facts are significant to either party. Moreover, this choice states that the buyer will prevail when, in fact, the buyer will not prevail due to the doctrine of estoppel by deed.
- A landlord owns a three-story building. The landlord leased the building and the surrounding property to a tenant for a term of six years, commencing on December 1, 2003 and terminating on November 30, 2009.
On April 15, 2004, a neighbor, who owned an adjoining parcel of land, was burning leaves and underbrush on his land. There was a high wind blowing in the direction of the land that the landlord leased to the tenant, and the wind carried the burning leaves onto the tenant’s property. The building caught fire and caused $150,000 in fire damages. Because of the fire, the tenant has not occupied the leased premises since the date of the fire because the building was rendered uninhabitable.
This state is a common law jurisdiction. In addition, the controlling Statute of Limitations for initiating an action for damages due to negligence is five years from the time the cause of damage occurs.
On May 1, 2004, the landlord brings suit against the tenant asserting breach of contract for failing to pay the rent. Judgment should be for
(A) the landlord, because a tenant remains liable to pay rent, even though as a result of the fire, the property was rendered uninhabitable.
(B) the landlord, because the covenant to pay rent runs with the land.
(C) the tenant, because it would be inequitable to enforce the rental provision in the lease because the premises was rendered uninhabitable.
(D) the tenant, because an increasing number of states have enacted statutes relieving a tenant of his obligation to pay rent where the premises are rendered uninhabitable by fire or other acts of God.
- (A) The tenant is still liable to the landlord for the monthly rental despite the fact that he can no longer reside in the building. At common law, the general rule is that a tenant remains liable to pay rent even though because of fire, floods, or other unforeseen action, the property is rendered totally uninhabitable. Please note that it is possible for the parties to provide in a lease for certain excuses for non-payment of rental; but in our hypothetical, no such provision was made. Choice (B) is incorrect. The covenant to pay rent does run with the land, making this a correct statement. However, this choice does not explain why the tenant is still required to pay rent even though the premises were so destroyed by fire as to cause the tenant to move out. Because this choice provides less of an explanation of why the landlord would prevail, it is a weaker choice, and choice (A) is a stronger reason why the landlord would prevail. Choice (C) is incorrect. Even though the premises were rendered uninhabitable, the tenant’s duty to pay rent is still absolute in spite of the destruction to the premises. Though it may be inequitable to compel the tenant to pay, that will not be part of the inquiry here. In this breach of contract situation, the tenant will be required to pay the rent and will not prevail. Choice (D) is incorrect. Even assuming it were true that an increasing number of states have enacted such statutes, this jurisdiction is not one of them. The facts state that this is a common law jurisdiction.
- A landlord owns a two-story building. The landlord leased the building and the surrounding property to a tenant for a term commencing on December 1, 2006 and terminating on November 30, 2008. Their leasehold agreement contained the following provisions: “The tenant covenants to pay the rent of $500 per month on the first day of each month and to keep the building situated upon leased premises in as good repair as it was at the time of said lease until the expiration thereof.”
On April 15, 2007, the tenant had her annual “Tax Time Party” where she and her friends would all walk to the post office together to mail their taxes and then have drinks and dinner at the tenant’s apartment to celebrate. After a few drinks, one of the tenant’s friends became belligerent toward one of the partygoers who challenged some of the friend’s deductions. The friend charged at the other partygoer and crashed into the wall after the partygoer got out of the way. The crash caused extensive damage to one of the walls in the apartment.
This state is a common law jurisdiction. In addition, the controlling Statute of Limitations for initiating an action for damages due to negligence is five years from the time the cause of damage occurs.
In an appropriate action to construe the tenant’s obligation under the covenant to keep the premises in repair, which of the following is the most accurate statement?
(A) The tenant is liable for normal wear and tear under the covenant to repair.
(B) The tenant is liable under such a covenant for all defects, including the damage to the wall.
(C) The tenant’s covenant to repair is void as against public policy, because the landlord is under an affirmative duty to make repairs on the demised premises.
(D) The tenant’s duty to keep the premises in good repair relieves the landlord of his obligation to disclose the existence of any latent defects.
- (B) The general rule provides that a tenant who covenants to keep the leasehold premises in good repair is liable under such a covenant for all defects regardless of their cause. As a result, the tenant remains liable for any defects that she caused or that were caused by third persons. Choice (A) is incorrect. At common law, such a covenant in a lease required the tenant to repair more than just the ordinary wear and tear of the premises. Because the covenant includes the promise to keep the premises in good repair, this would include the duty to make the premises the same as they were the day that the landlord delivered them to the tenant. This includes fixing the wall and not just ordinary wear and tear. Choice (C) is incorrect. At common law, the tenant is responsible for such damage when the tenant makes such a covenant. It would then be irrelevant for the tenant to raise the issue of the covenant beingvoid as against public policy because the landlord was never under any such duty to fix the wall. Harsh as the common law rule might be, this argument will still be unpersuasive. Choice (D) is incorrect. A landlord may be liable in tort to the tenant, her guests, licensees, and invitees if, at the commencement of the Lease, there is a dangerous condition that the landlord knows or should know about and the discovery of which would not likely occur by the tenant exercising due care. The tenant’s taking on this covenant does not relieve the landlord of this obligation, so this statement is incorrect. Moreover, this choice focuses on the wrong issue, making it incorrect.
- A farmer, being fee simple owner of a farm, devised it to his niece and her husband as tenants by the entirety. The niece and her husband took immediate possession of the farm and lived there with their daughter and son. Thereafter, the husband died in an automobile accident. In 1972, two years after her father’s death, the daughter moved to another state.
The son lived with his mother on the farm until her death intestate in 1980. The son continued in exclusive possession of the farm until his death in 2001. In his will, the son devised the farm to a local charity. When he was alive, the son was unaware that his sister was still alive and that title to the farm had descended to the two of them as their mother’s sole surviving heirs. Since his mother’s death in 1980, the son has held himself out as the owner of the farm, maintaining it and paying all of the taxes on the property. The sister had not communicated with either her mother or her brother since her redomiciling in 1972. The jurisdiction in which the farm is located has a 20-year limitation period for the acquisition of property by adverse possession.
What interest, if any, does the sister have in the property?
(A) None, because of her own laches.
(B) None, because the brother acquired title to the farm by adverse possession.
(C) An undivided one-half interest because the brother’s possession was not adverse to her title.
(D) An undivided one-half interest because the 20-year limitation period did not run against her because she was unaware of the brother’s exclusive possession.
- (C) Here, the brother and sister acquired title to the farm as tenants in common (by descent from their mother). In a tenancy in common, each tenant owns an undivided fractional part of the property and none owns the whole (as in joint tenancy). The brother will not own the property outright because the brother’s conduct never amounted to an ouster of his sister. An ouster occurs where one co-tenant manages to wrongfully exclude her co-tenants from possession of the property. When one co-tenant ousts from possession her co-tenant, the ousted tenant has a cause of action against the possessor, not to put her out, but to regain possession for herself with the possessor. Where one co-tenant stakes a claim for exclusive possession of any part of the property, the claim alone may amount to an ouster. The brother’s living on the property alone will not be enough to oust the sister of her undivided one-half interest because the brother’s possession was not adverse to his sister’s title. Choice (A) is incorrect. Laches is a defense to any action brought in equity. If a party had waited an unreasonably long period of time in which to bring a claim, the other party may argue that he has relied upon that lack of assertion of rights to his detriment. However, nothing in the facts suggests that the brother has, in fact, relied upon his sister’s failure to enforce her rights to the farm, which has led to his detriment. Choice (B) is incorrect. The brother’s possession of the farm was not hostile and adverse because he never, in fact, ousted his sister. Choice (D) is incorrect. This falsely suggests that the 20-year period would have started running if the sister became aware of her brother’s exclusive possession or somehow learned of it. However, the brother’s exclusive possession would still not start the clock running if there had been no affirmative ouster of the sister. Because this choice focuses too much on the sister’s awareness and not on the brother’s conduct, it is incorrect.
- A landlord is the owner in fee simple of a tract of land on which is situated a large office building. The landlord leases the land and building thereon to a tenant for a term of seven years, commencing on August 1, 2001, and terminating at midnight on July 31, 2008. The lease contains the following provisions
“(1) The tenant covenants to keep the building on the leased premises insured against fire in the amount of $100,000.”
After three years of the lease had expired, the tenant assigned the entire balance of the lease period to a doctor, who took immediate possession of the leased property. Then in 2005, the doctor assigned his leasehold interest to a dentist. The dentist went into possession, but failed to pay any rent for two months. Soon afterwards, a fire spread from an adjoining building to the leased property, completely destroying the building thereon.
While the tenant was in possession of the leased property, he carried a fire insurance policy on the premises in the amount of $100,000. However, the tenant allowed the policy to lapse after his assignment to the doctor. The doctor did carry insurance on the leased building, but only in the amount of $50,000. When the dentist took possession of the building, he did not obtain any fire insurance.
The landlord learned that the building was not insured at the time of the fire as per the original agreement. In an action by the landlord against the tenant to recover for the fire loss, the landlord will most probably
(A) recover, because in accordance with the Rule in Spencer’s Case, the covenant to maintain fire insurance would “touch and concern” the land.
(B) recover, because the tenant’s obligation to maintain fire insurance did not terminate after his assignment to the doctor.
(C) not recover, because the covenant to provide fire insurance did not run with the land.
(D) not recover, because the dentist, as assignee, would be liable for the fire loss.
- (B) A covenant in a lease to pay (fire) insurance is held to “run with the land.” In this regard, a covenant to pay insurance is capable of running if and only if the landlord is bound to use the proceeds for repair or replacement. It is important to understand that the tenant’s assignment of his leasehold interest to the doctor did not relieve or extinguish the tenant’s contractual obligations under his lease with the landlord. In short, an assignment does not release the tenant from his contract obligations to the landlord under the terms of this original leasehold agreement. To be sure, a lease is a contract as well as a conveyance. Choice (A) is incorrect. The Rule in Spencer’s Case provides that an assignee of the leasehold estate cannot be held liable for breach of covenant if the covenant is of a type that “does not touch and concern the land.” However, the tenant still has a contractual obligation to the landlord to buy proper fire insurance based on the original covenant in the lease. So, even though this covenant will run to successive assignees, the tenant can still be liable. This choice focuses more on why successive assignees would be liable, but does not explain why the tenant is still liable. Thus, it is not as complete as choice (B). Choice (C) is incorrect. Covenants to insure the buildings on leased premises do, in fact, run with the land. Because this choice states otherwise, it is incorrect. Choice (D) is incorrect. The dentist may be liable in his own right because the covenant does run with the land. However, in stating that the landlord will not recover against the tenant, this choice suggests that the tenant is not liable for the failure to insure properly. Because the tenant is still liable under the original lease, the landlord will be able to recover against the tenant.
- A homeowner gave his friend a mortgage on his property to secure a loan from the friend to the homeowner in the amount of $100,000. The homeowner then gives an investor a mortgage on the property to secure a loan from the investor to the homeowner in the amount of $80,000. The friend then records his mortgage. The homeowner then gives a bank a mortgage on the property to secure a loan from the bank to the homeowner in the amount of $50,000. The investor then records. The bank then records.
The homeowner lost his job and was unable to pay the mortgages on the property. In a foreclosure proceeding in which the friend, the investor, and the bank were parties, the property sold for $160,000. The recording statute in this jurisdiction provided “unless recorded, all written instruments affecting title to land are void as to subsequent purchasers who paid value and without notice.”
What dollar amount, if any, should go to the friend?
(A) $0.
(B) $30,000.
(C) $80,000.
(D) $100,000.
- (C) A mortgage is defined as an interest in land created by a written instrument providing security for the performance of a duty or the payment of a debt. A mortgage should be and is generally recorded. A foreclosure proceeding is the method by which the security (i.e., the property), or proceeds for the sale thereof, is applied to the satisfaction of the debt. A mortgagee has both an in personam claim against the mortgagor and an in rem action against the security. In these facts, all three mortgagees (the friend, the investor, and the bank) are involved in a foreclosure proceeding for the proceeds of the sale. Priority as to these proceeds will be determined by such factors as order of recordjng and notice of prior mortgages, depending on the applicable recording statutes in the jurisdiction. In a pure notice jurisdiction, a subsequent bonafide purchaser is protected if the mortgagee of a prior mortgage has not recorded at the time when the subsequent bonafide purchaser takes his mortgage, regardless of who records first. Here, the bank took its mortgage after the friend had already recorded. Therefore, the bank has notice and will not prevail. The investor will prevail, however, because she was the last BFP without notice of any prior recordation, because at the time the investor took her mortgage, the friend had not yet recorded. The investor will then get $80,000 from the proceeds to satisfy her debt, leaving $80,000 from the sale price of $160,000. Although $80,000 will not fully satisfy the friend’s interest, he will be entitled to receive the rest of proceeds and the bank takes nothing. Choice (A) is therefore incorrect, because it is the bank that will take nothing. Choice (B) is incorrect. The only apparent significance of this dollar amount is that it represents the difference between the investor’s mortgage (of $80?000) and the bank’s mortgage (of $50,000). The friend would be entitled to the rest of the purchase price after the investor is fully compensated. There is no reason why the friend would be limited to only $30,000 of the remaining $80,000. Choice (D) is incorrect. The property sold for $160,000. Because the investor is the last BFP to take without notice, she will be paid first from the sale proceeds. Because this leaves only $80,000 remaining, that is the most that the friend can receive. The friend is not entitled to receive the full $100,000 as stated in this choice because the friend will not take first. As a result, his debt will not be fully satisfied.
- A landlord is the owner in fee simple of a tract of land on which is situated a large office building. The landlord leases the land and building thereon to a tenant for a term of seven years, commencing on August 1, 2001, and terminating at midnight on July 31, 2008. The lease contains the following provisions:
“(1) The tenant covenants to pay the rent of $750 per month on the first day of each month.”
After three years of the lease had expired, the tenant assigned the entire balance of the lease period to an attorney, who took immediate possession of the leased property. Then in 2005, the attorney assigned his leasehold interest to a doctor. The doctor went into possession, but failed to pay any rent for two months.
After the doctor failed to make his rental payments for the first two months of his lease, the landlord brought suit against the tenant to recover for the unpaid rent.
Judgment should be for
(A) the landlord, because the tenant’s contractual obligation under the lease survived the assignments.
(B) the landlord, because he did not object to the assignments.
(C) the tenant, because the doctor, as assignee, would only be held liable.
(D) the tenant, because his assignment to the attorney constituted a novation, thereby extinguishing his obligation to pay rent.
- (A) Where a tenant—assignor transfers all of his leasehold interest to an assignee, such assignment does not release the original tenant from his contract obligations to the landlord under the terms of the lease. This is true even when the assignee, thereafter, assigns/transfers his leasehold interest to a sub-assignee. Choice (B) is incorrect. The landlord prevails here not because he didn’t object to the assignments, but rather because the tenant is still primarily liable under the terms of the original lease. Because privity of contract still exists between the tenant and the landlord under the original lease, the landlord can bring suit against the tenant. Choice (C) is incorrect. Where a tenant—assignor transfers all of his leasehold interest to an assignee, such assignment does not release the original tenant from his contract obligations to the landlord under the terms of the lease. Choice (D) is incorrect. The assignment to the attorney did not constitute a novation, but rather an assignment of the balance of the leasehold interest. A novation is a complete substitution of one party for another in a contract. An assignment alone will not effectuate a complete substitution of parties. Without express language that the substitution of the attorney would completely discharge the tenant from all prior duties, the tenant still remains liable for the rent under the original lease agreement.
- A landlord is the owner in fee simple of a tract of
land on which is situated a studio. The landlord
leases the land and building thereon to a tenant for
a term of seven years, commencing on August 1,
2001, and terminating at midnight on July 31, 2008.
The lease contains the following provisions:
“(1) The tenant covenants not to assign the leased premises without the consent of the landlord.”
After three years of the lease had expired, the tenant, without the consent of the landlord, assigned the entire balance of the lease period to a dance instructor, who took immediate possession of the leased property. The landlord accepted rental payments from the dance instructor. Then in 2005, the dance instructor assigned his leasehold interest to a yoga instructor who went into possession.
Assume for the purposes of this question only that the landlord, after learning of the dance instructor’s assignment to the yoga instructor, brings suit against the dance instructor to have the assignment declared
void.
The landlord will most likely
(A) succeed, because the original leasehold agreement prohibited assignments.
(B) succeed, because the covenant prohibiting assignments did not run with the land.
(C) not succeed, because in accordance with the Rule in Dumpor’s Case, where the landlord consents to one transfer, he waives his right to avoid future transfers.
(D) not succeed, because the privity of estate between the dance instructor and the landlord terminated when he assigned the leasehold to the yoga instructor.
- (C) In accordance with the Rule in Dumpor’s Case, if a landlord grants consent to one transfer (e.g., the tenant’s assignment to the dance instructor), he waives his right to avoid future transfers, assigns, or subleases in violation of a prohibition in the lease against such transfers. By the same token, if the lessee/assignee pays rent to the landlord and such rent is accepted by him, in spite of the fact that the transfer was in violation of the lease, the landlord will be deemed to have waived his right to avoid the transfer. Choice (A) is incorrect. Certainly, the original provision limited the ability to assign. However, when the landlord accepted rent from the dance instructor, the assignee, he waived any right he had to enforce it against any subsequent assignments. Because this choice fails to account forthe fact that this prohibition was waived, it is incorrect. Choice (B) is incorrect. Covenants not to assign or sublease the premises have been held to run with the land. Because this choice states that such covenants do not, it is incorrect. Choice (D) is incorrect. While this is a true statement, this does not explain why the landlord is no longer in a position to contest the transfer to the yoga instructor. The landlord Loses because he waived the prohibition on assignments by accepting rent from the dance instructor, not because there is no Longer privity of estate between the two parties.
- A landlord, the owner of a two-story dwelling house, leased it completely furnished to a tenant for a 1 0-year period. Toward the end of the seventh year of the term, a violent storm blew off several of the roof shingles. The tenant noticed a leak through the roof where the shingles had blown off, but didn’t make any repairs. A month later, a severe rain storm occurred, and water leaked through the roof, causing damage to the valuable parquet floors in the two rooms below. Before the term of his lease ended, the tenant discovered that the landlord had not paid his taxes on the property, so he purchased the property through a sheriff’s sale. The tenant refuses to make any further rental payments on the property.
If the landlord brings suit against the tenant for payment of rent in arrears, he would most likely
(A) succeed, because the tenant is estopped to deny the landlord’s title.
(B) succeed, because of his security interest in the property.
(C) fail, because the purchase by the tenant vitiated any further contractual obligations.
(D) fail, because the landlord was under a duty to keep the demised premises in reasonably good repair.
- (C) After purchasing the property at the sheriff’s sale, the tenant, as record title owner, would no longer remain obligated under the leasehold contract. As such, choice (C) is correct because the sheriff’s sale would vitiate the tenant’s obligation to make any further rental payments to the Landlord. Choice (A) is incorrect. The landLord cannot successfully argue that the tenant is estopped to deny the landlord’s title because the landlord no Longer has title. Furthermore, this choice is incorrect in stating that the landlord will prevail. Choice (B) is incorrect. The landlord will not be able to claim a security interest in the property because the landlord does not hold a lien on the property. Choice (D) is incorrect. This choice falsely suggests that the landlord’s failure to keep the premises in good repair was the reason the tenant no longer owes rent. This choice disregards the sheriff’s sale to the tenant, which is the best reason why the tenant is no longer obligated to pay rent.
- A landlord was the owner of a two-story dwelling house and leased it completely furnished to a tenant for a 10-year period. Toward the end of the seventh year of the term, a violent storm blew off several of the roof shingles. The tenant noticed a leak through the roof where the shingles had blown off, but didn’t make any repairs. A month later, a severe rain storm occurred and water leaked through the roof, causing damage to the valuable parquet floors in the two rooms below.
In a subsequent proceeding by the landlord against the tenant to recover for the damage to the parquet floors resulting from the leak, liability would most likely be imposed upon
(A) the landlord, because he was under an implied obligation to keep the premises in a habitable condition.
(B) the landlord, because he was under an affirmative obligation to deliver the premises in a reasonable state of repair.
(C) the landlord, because of the contractual obligation under the lease to make all necessary repairs during the term of the lease.
(D) the tenant, because a tenant for years is obligated to make such ordinary repairs on the leased property.
- (D) The tenant is liable for the damage to the floors caused by his failure to replace the shingles because a tenant for years is legally bound to make such ordinary repairs on the leased property as will avoid serious injury to the property. Thus, the tenant’s failure to make the necessary repairs would make him liable for such permissive waste. Choice (A) is incorrect. A landlord impliedly covenants in the lease not to interfere with the quiet enjoyment of the premises. One way that that quiet enjoyment can be interfered with is if the landlord engages in behavior (or fails to engage in certain behavior) that amounts to a constructive eviction. However, the living arrangement must be so severe as to cause the premises to be uninhabitable such that the tenant must move out. Here, the roof leaked from a hole caused by several of the shingles falling away. This caused water damage to the parquet floors. However, nothing in the facts suggests that the damage was so severe as to cause heavy flooding or some other condition that would make the premises uninhabitable. So, although the landlord may have an obligation to keep the premises in a habitable condition, nothing about the damage here suggests that the Landlord is somehow failing to provide habitable premises. Choice (B) is incorrect. Even though the landlord was required to deliver the possession of the premises and was required to not interfere with the tenant’ quiet enjoyment, the tenant would be under a duty to make ordinary repairs. Choice (C) is incorrect. Nothing in the facts suggests that there was an express contractual obligation for the landlord to make all necessary repairs during the term of the lease. Absent those provisions, it is the tenant who has the duty not to commit permissive waste on the premises.
- A landlord was the owner of a two-story dwelling house and leased it completely furnished to a tenant for a 10-year period. Two years after the tenant entered into the leasehold agreement with the landlord, he subleases the house to his cousin. Thereafter, the cousin defaults on the rental payments.
If the landlord brings an action to recover the past rent due from the cousin, the landlord will:
(A) recover, because pnvity of estate exists between a landlord and sublessee.
(B) recover, because there is privity of contract between a landlord and sublessee.
(C) recover, because there is both privity of estate and privity of contract between a landlord and sublessee.
(D) not recover, because privity of estate does not exist between the landlord and sublessee.
- (D) The landlord may not recover for rent due from sublessee because there is no privity of estate between landlord and sublessee. The landlord’s only cause of action remains against the tenant under their original leasehold agreement. Choice (A) is incorrect. This is an incorrect statement. There is no privity of estate between a landlord and a sublessee. Choice (B) is incorrect. This is an incorrect statement. There is no privity of contract between a landlord and a sublessee. The landlord is in privity of contract with the tenant. Choice (C) is incorrect. This is an incorrect statement. There is neither privity of estate nor privity of contract between a landlord and a sublessee.
- A corporation, under the authority of a statute of the state, sued to have condemned 1,000 acres of forested land owned by a timber company, which it planned to develop for use as a state recreational area and state game lands. After a hearing, the state court ordered possession of the land surrendered to the corporation, prior to determination of compensation, upon deposit in court of a sum deemed adequate to cover damages that might be awarded. The timber company immediately commenced an action to enjoin the court-ordered sale of its property.
Which of the following would be the best ground for upholding the state court’s order?
(A) The power of eminent domain may only be delegated directly to a private enterprise for a public related use or activity.
(B) The power of eminent domain may only be delegated to a public authority through a legislative determination.
(C) The injured party has not proved such irreparable injury to use as amounts to a “taking.”
(D) The Fifth Amendment’s power of eminent domain, incorporated by the Fourteenth Amendment as applicable to the states, does not require that payment be made prior to condemnation of the property.
- (A) The power of eminent domain may be delegated directly or indirectly to a private person or enterprise subject to the requirements that the taking be (a) for a public use, and (b) just compensation be given. Choice (B) is incorrect. The court will not want to rely on this argument because it is not on point with the facts addressed in the fact pattern. One of the issues before the court would be whether or not a private enterprise can be delegated the power of eminent domain. Choice (B) addresses the issue of how power can be delegated to a public authority. However, the corporation is the party trying to condemn the forestland, not a public authority. Thus, the argument in (B) isa less persuasive reason for allowing the land to be surrendered to the corporation. Choice (C) is incorrect. It is a weak argument for the court to argue that the party was not irreparably injured enough and thus no taking occurred. One way that a taking ocurs is when the government confiscates or physically occupies property. When there is this kind of conduct, there is a taking and it is not necessary for the dispossessed ownerto show that she was irreparably injured. Because this is not part of the court’s inquiry in deciding whether or not the land should be surrendered to the corporation, it cannot be the basis or best grounds for upholding the order. Choice (D) is incorrect. As long as just compensation is given to the timber company, it will not matter whether that compensation was given prior to condemnation or afterwards. This will not be the best reason to uphold the court’s decision here.
- A state on the eastern seaboard has a long stretch of scenic, ecologically unique, and relatively unspoiled seacoast. Most of the coastline is privately owned by persons who have permanent or vacation residences on their coastal lands. A man purchased a two-acre tract of littoral (i.e., coastal) land in the state. When the man bought the property, there was a narrow concrete walkway (about 20 feet in length and 5 feet in width) adjoining the land. The walkway, which was designed to provide pedestrian access to the beach, was owned by the State Coastal Commission. When the man moved into his beach house, he noticed that the walkway was rarely used. As a result, the man decided to enclose the walkway and construct a paddle ball court over the entire area. He did so without notifying the State Coastal Commission. He has continued to use this area in excess of the period required for adverse possession.
In an appropriate action brought by the man to establish title to the walkway area, which of the following must he establish if he is to prevail?
(A) Real property interests can be abandoned by a governmental entity without an official vote.
(B) Lack of use of the walkway by the state created an irrevocable license in him.
(C) Real property interests can be lost by a state by adverse possession.
(D) Due to a defect in the chain of title, the state does not own this piece of land.
- (D) Because title togovernment-owned property cannot be gained through adverse possession, the man’s best argument would be that someone other than the state actually owns the property. Choice (A) is incorrect. This is a weak choice because it only explains that the government has abandoned the property interest without giving a reason as to why or how that interest could have been abandoned. The man would not want to only argue that the property interest was abandoned; he would want to argue why he feels that interest was lost, and this choice does not supply the reasons. Choice (B) is incorrect. The man is trying to establish title to the walkway, i.e., ownership in the strip of land. This choice discusses licenses, which at most give the man the right to use the walkway, and this is not what the man is trying to establish. Choice (C) is incorrect. This statement goes against the gen eral rule that title to government-owned property cannot be gained through adverse possession, making it a very weak argument.
- A landlord, an owner in fee simple, leased a three- story house to a tenant for a term of 10 years. By the terms of the lease, the tenant expressly covenanted to pay a monthly rental of $300 and to pay the taxes on the premises during the term of the lease. The lease also stipulated that the tenant, as lessee, may not assign or sublease the said premises.
The tenant and his family lived in the house for two years. Then the tenant, still owing the landlord three months back rent, assigned his leasehold interest in the property to a student, who was unaware of the prohibition against assignments. This written assignment expressly provided that the student would pay the landlord the monthly rental of $300, but was silent concerning the payment of taxes. The landlord never objected to this assignment. Six months after the assignment, the landlord conveyed his property interest in the premises to a new owner.
The student failed to pay the taxes on the property. The new owner, after paying the taxes, brought suit against the student for the amount paid. Judgment should be for
(A) the new owner.
(B) the student, because he did not contract with the tenant to pay the taxes on the property.
(C) the student, because the covenant to pay taxes will not “run with the land,” unless the intent is clearly expressed in the original lease.
(D) the student, because the covenant to pay taxes is merely collateral and does not “run with the land.”
- (A) By virtue of the assignment, the assignee (i.e., the student) falls into privity of estate with the landlord, and he is liable on all covenants in the original lease that “run with the land.” Because the covenant to pay taxes (under the original leasehold agreement between landlord and tenant) “runs with the land,” the student, as assignee, would be held liable. Choice (B) is incorrect. Even though the student did not specifically contract with the tenant in the facts, the fact remains that the covenant did run with the land. Because the student is an assignee of the tenant’s property interest, the student is now in privity of estate with the new owner. Because of this, the student is bound by the covenant, even though there was no separate contract with the tenant to pay the taxes. Choice (C) is incorrect. Even though the Lease does not in express language say that the covenant will run with the land, the lease does say that the taxes will be paid during the term of lease. So, there is an intention that the covenant to pay rent would run with land in spite of a lack of express language. Because this answer choice falsely suggests that a Lack of express language leads to the student not being liable, it is incorrect. Choice (D) is incorrect. This type of covenant is not collateral and would run with the land. Because this answer choice states the opposite, it is incorrect.
- A homeowner owns a home in fee simple and leased it to his cousin for a term of 10 years. By the terms of the lease, the cousin expressly covenanted to pay a monthly rental of $300 and to pay the taxes on the premises during the term of the lease. The lease also stipulated that the cousin, as lessee, may not assign or sublease the said premises.
The cousin and his family lived in the house for two years. Then the cousin, still owing the homeowner three months back rent, assigned his leasehold interest in the property to a friend, who was unaware of the prohibition against assignments. This written assignment expressly provided that the friend would pay the homeowner the monthly rental of $300, but was silent concerning the payment of taxes.
After the cousin’s assignment to the friend, the homeowner sues the cousin to recover the rent in arrears. The homeowner will most likely
(A) succeed, because the homeowner and the cousin stood in privity of contract.
(B) succeed, even though the homeowner had notice of the assignment.
(C) not succeed, because the cousin had assigned his interest in the premises before the homeowner brought suit.
(D) not succeed, because he did not object to the assignment.
- (A) Under the terms of their original leasehold agreement, the cousin would remain Liable for the rent in arrears. The cousin’s assignment to the friend would not operate as an extinguishment of his duty to pay rent to the homeowner under the terms of their original agreement. An assignment does not release the tenant from his contractual obligation to the Landlord under the lease. To be sure, a lease is a contract as well as a conveyance. Choice B) is incorrect. This choice is erroneous because it falsely suggests that the homeowner’s notice (of the assignment) is somehow relevant to the cousin’s obligation to pay. The cousin’s obligation to pay is based upon the privity of contract that arose from the original lease agreement, and the homeowner’s notice (or lack thereoO will have no effect upon this original obligation. Choice (C) is incorrect. Even though the cousin had already assigned before the homeowner brought suit, the cousin is still liable for the rent under the original agreement based on privity of contract. The homeowner’s right to collect the rent is not extinguished because he brings suit after the assignment. Choice (D) is incorrect. Whether the homeowner objected to the assignment or not, the fact remains that the cousin is still liable for the rent. This obligation to pay the rent is not dependent upon the homeowner’s consent or objection to the assignment.
- A landlord owned a three-story home in fee simple and leased it to his friend for a term of 10 years. By the terms of the lease, the friend expressly covenanted to pay a monthly rental of $300 and to pay the taxes on the premises during the term of the lease. The lease also stipulated that the friend, as lessee, may not assign or sublease the said premises.
The friend and his family lived in the house for two years. Then the friend, still owing the landlord three months back rent, assigned his leasehold interest in the property to a professor, who was unaware of the prohibition against assignments. This written assignment expressly provided that the professor would pay the landlord the monthly rental of $300, but was silent concerning the payment of taxes. The landlord never objected to this assignment. Six months after the assignment, the landlord conveyed his property interest in the premises to a new owner.
After residing in the house for a year, the professor subleased the third floor to a student for a term of two years; the agreement provided that the professor would receive a monthly rental payment of $100.
After the student had made regular rental payments to the professor for the first six months of the sublease, he defaulted. The following month, the professor sent the new owner $200 as payment for rent and notified him that he should collect the remaining $100 from the student, who refused to vacate the premises. In an action by the new owner against the student to recover the additional $100 for rent due, he will most probably
(A) recover, because the landlord and sublessee are in privity of estate.
(B) recover, even though he never objected to the sublease.
(C) not recover, because the professor would only be held liable.
(D) not recover, because the professor remains liable.
- (D) Because the sublessee (the student) is not in privity of estate with the landlord, the sublessee is not subject to the covenants or terms of the leasehold agreement. Therefore, the professor will remain liable for the entire $300 rental. Choice (A) is incorrect. The new owner will not recover because the landlord and sublessee are neither in privity of contract nor privity of estate. Choice (B) is incorrect. Whether the new owner objected to the sublease or not, the new owner still cannot recover from the student because there is neither privity of estate nor privity of contract here. Choice (C) is incorrect. The professor will be liable as he is in privity of estate with the landlord as an assignee. The friend will still remain in privity of contract with the landlord based upon the original lease agreement. Because this answer choice states that only the professor is liable, it is incorrect.
- A property owner who owner her property in fee simple leased a three-story house and the 21 -acre tract upon which it stood to a friend for a term of 10 years. By the terms of the lease, the friend expressly covenanted to pay a monthly rental of $300 and to pay the taxes on the premises during the term of the lease. The lease also stipulated that the friend, as lessee, may not assign or sublease the said premises.
The friend and his family lived in the house for two years. Then the friend assigned his leasehold interest in the property to a cousin, who was unaware of the prohibition against assignments. This written assignment expressly provided that the cousin would pay the owner the monthly rental of $300, but was silent concerning the payment of taxes. The owner never objected to this assignment. Six months after the assignment, the owner conveyed her property interest in the premises to a developer.
During the cousin’s tenancy, the State Highway Authority filed proceedings to condemn two acres of the tract for the purpose of highway construction. As a result, the cousin contacted the developer and informed him that he should be entitled to an apportioned reduction in the rental. After the developer refused to reduce the rent, the cousin brings an action to have his rent apportioned pro tanto.
Judgment should be for
(A) the developer, although the cousin would be entitled to terminate the lease.
(B) the developer, because the cousin would be held to the original leasehold contract.
(C) the cousin, because the value of his leasehold interest was reduced pro tanto.
(D) the cousin, because eminent domain operates as an apportionment of rent.
- (B) The question of law presented here is, namely, when there is a partial taking of a leasehold estate by condemnation and the remaining portion is susceptible for occupation, shall the contractual monthly rental be abated pro tanto for the part taken during the remainder of the time of the lease? The majority rule is that rentals are not abated in this situation; rather, the tenant is obligated to continue the payment of the rentals provided in the lease contract and must look to an apportionment of the damages assessed against the condemning authority based on the reduced value of his lease. Choice (A) is incorrect. If only a portion of the Leased property is condemned, the tenant is not discharged from her obligation to pay rent. Because the facts state that only two acres of the 21-acre tract are being condemned, the cousin will not be allowed to terminate the lease. Choice (C) is incorrect. The majority rule is that rentals are not abated in this situation; rather, the tenant is obligated to continue the payment of the rentals provided in the lease contract and must look to an apportionment of the damages assessed against the condemning authority based on the reduced value of his lease. Choice (D) is incorrect. Eminent domain itself does not necessarily operate as an apportionment of rent, but instead allows the tenant to a share of the condemnation award. Where, as here, a portion of the land is condemned and the other portion is suitable for occupation, the tenant is obligated to continue the payment of the rent as provided in the lease and is entitled to a share of the condemnation award.
- A landowner owned a large building in the city. On January 15, the landowner leased the building to a tenant for a period of 20 years at a rental of $10,000 per month. The leasehold agreement between the landowner and tenant provided that the latter was not permitted “to assign this lease to anyone except a corporation with an ‘A’ credit rating from a well- known credit rating company.” On February 1 of the following year, the tenant leased the premises to a corporation that did not have the required credit rating. The tenant—corporation lease was for a period of five years, with a rental of $15,000 per month, payable by the corporation to the tenant. In addition, the corporation agreed to abide “by all of the terms and conditions of the lease between the landowner and tenant.”
Which of the following accurately states the legal effect of the non-assignability clause contained in the landowner—tenant leasehold contract?
(A) The non-assignability provision had no legal effect.
(B) The non-assignability provision made the assignment from the tenant to the corporation ineffective.
(C) The tenant-corporation lease did not effectuate a breach of the landowner—tenant contract.
(D) Although the tenant-corporation lease constituted a breach of the landowner—tenant contract, the landowner would nevertheless be required to recognize the validity of the transfer (of the premises) to the corporation.
- (A) Choice (A) is the most accurate statement. The non-assignability provision had no legal effect. Because the tenant’s transfer (of the premises) to the corporation constituted a subLease, not an assignment, the said provision would have no legal effect. Students must know the distinction between an assignment and a sublease. An assignment is a transfer of a tenant’s entire leasehold estate to another; whereas a sublease is a transfer of anything less than a tenant’s entire leasehold estate. Choice (B) is incorrect. Because the tenant’s transfer (of the premises) to the corporation constituted a sublease, not an assignment, the said provision would have no legal effect. This answer choice is incorrect in referring to the transfer as an assignment when it was a sublease. Also, the transfer is, in fact, effective because the provision is inapplicable. Choice (C) is less correct because, although factually true, it does not answer the question asked. The question asks about whether the assignment clause has any effect on the validity of the transfer between the tenant and the corporation, not whether or not there was a breach of the original contract. Therefore, choice (C) is not the most accurate sttement and thus incorrect. Choice (D) is incorrect. This choice falsely states that the transfer was a breach of the original contract. Because the contract said nothing about subleases, there was never a breach of the original agreement.
- A landlord owned a large building in the city. On January iS, the landlord leased the building to a
tenant for a period of 20 years at a rental of $10,000 per month. The leasehold agreement between the landlord and tenant provided that the latter was not permitted “to assign this lease to anyone except a corporation with an ‘A’ credit rating from a well- known credit rating company.” On February 1 of the following year, the tenant leased the premises to a corporation that did not have the required credit rating. The tenant—corporation lease was for a period of five years, with a rental of $15,000 per month, payable by the corporation to the tenant. In addition, the corporation agreed to abide “by all of the terms and conditions of the lease between the landlord and the tenant.”
One year later, the corporation leased the premises to a health club for the balance of the term of the corporation—tenant lease. The health club owner took possession of the said premises on February 1, the same day that the corporation vacated its occupancy. Pursuant to the corporation—health club leasehold agreement, the latter was obligated to pay a monthly rental of $17,500 directly to the corporation. The health club has a ‘B’ credit rating with the well-known credit rating company. For one year, the health club paid $17,500, each month directly to the corporation. During that same period, the corporation continued to pay $15,000 each month to the tenant, while the latter paid $10,000 (each month) to the landlord. The landlord knew about the leases to the corporation and the health club and protested promptly, but took no further action, apparently satisfied as long as he received his $10,000 per month from the tenant.
Thereafter, the health club abandoned the premises and stopped paying rent to the corporation. After the health club discontinued paying rent, the corporation stopped paying rent to the tenant. When the tenant failed to receive his rent, he, too, stopped paying rent to the landlord.
If the landlord brings suit to recover for past rent due, which of the following is most accurate?
(A) The landlord may recover against the tenant for past rent due.
(B) The landlord may recover against the corporation for past rent due.
(C) The landlord may recover against the health club for past rent due.
(D) The landlord may recover against the corporation and the health club, jointly and severally,
for past rent due.
- (A) In the case of a subLease, privity of estate and privity of contract exist between the original landlord and tenant. Similarly, privity of estate and privity of contract exist between the tenant and subtenant. However, neither privity of contract nor privity of estate exists between the original landlord and subtenant. Therefore, the Landlord cannot sue the subtenant. As a consequence, the Landlord may not recover against subtenants (the corporation and the heaLth club owner); he may recover only against the tenant under the terms of their original leasehold contract. Choice (B) is incorrect. Neither privity of contract nor privity of estate exist between the original landlord and subtenant. Therefore, the landlord cannot sue the corporation subtenant. Choice (C) is incorrect. Neither privity of contract nor privity of estate exist between the original landlord and subtenant. Therefore, the landlord cannot sue the subtenant health club owner. Choice (D) is incorrect. Neither privity of contract nor privity of estate exists between the originaL landlord and subtenant. Therefore, the landlord cannot sue the subtenants jointly and severally.
- A property owner owned a large building in the city. The owner leased the building to a shopkeeper for a period of 20 years at a rental of $10,000 per month. The leasehold agreement between property owner and shopkeeper provided that the latter was not permitted “to assign this lease to anyone except a corporation with an ‘A’ credit rating from a well- known credit rating company.” One month later, the shopkeeper leased the premises to a corporation that did not have the required credit rating. The shopkeeper-corporation lease was for a period of five years, with a rental of $15,000 per month, payable by the corporation to the shopkeeper. In addition, the corporation agreed to abide “by all of the terms and conditions of the lease between the property owner and the shopkeeper.”
One year later, the corporation leased the premises to a non-profit organization for the balance of the term of the corporation—shopkeeper lease. The non-profit organization took possession of the said premises the same day that the corporation vacated its occupancy. Pursuant to the corporation—nonprofit leasehold agreement, the latter was obligated to pay a monthly rental of $17,500 directly to the corporation. The non-profit organization has a ‘B’ credit rating with the well-known credit rating company.
Which of the following is not correct regarding the legal relationship(s) of the various parties?
(A) Privity of estate and privity of contract exist between the property owner and the shopkeeper.
(B) Privily of estate and privity of contract exist between the shopkeeper and the corporation.
(C) Privily of estate, but not privily of contract, exists between property owner and the corporation.
(D) Neither privily of estate nor privily of contract exists between the property owner and the nonprofit organization.
- (C) This is a false statement and thus the correct choice. The statement is incorrect in suggesting that the property owner and the corporation are in privity of estate, which is false; the landlord and subtenant are never in privity of estate. Choice (A) is incorrect. This is a true statement because there is privity of estate and privity of contract between the landlord and the tenant. Likewise, privity of estate and privity of contract exist between the tenant and subtenant. However, neither privity of contract nor privity of estate exists between the landlord and the subtenant. Choice (B) is incorrect. This is a true statement because privity of estate and privity of contract exist between the tenant and the subtenant. Choice (D) is incorrect. This is a true statement because as between the property owner and the nonprofit organization, there is neither privity of estate nor privity of contract.
- A landlord was the owner of a vacant warehouse. On May 1, the landlord leased the warehouse to the tenant for a term of five years with a renewable option. The leasehold agreement provided in part:
“The tenant hereby acknowledges that certain areas of the leased premises are in a state of disrepair and unsafe for the conduct of business. Nonetheless, the tenant agrees to assume full responsibility for the necessary repairs. Furthermore, the tenant agrees to indemnify the landlord for any loss resulting from the condition of the said premises.”
Under the terms of the lease, the landlord delivered possession of the warehouse to the tenant on May 2. On that date, an employee of the tenant was working in the warehouse. He was moving some equipment into the warehouse when a section of the ceiling collapsed and fell on him. At the time the landlord and the tenant entered into their lease, only the landlord was aware that the ceiling was defective and needed to be repaired. Although the landlord could have remedied the defect before delivering possession to the tenant, the landlord didn’t perform any repair work on the ceiling. The employee initiated suit against the landlord to recover damages resulting from his injury. There are no applicable statutes.
If the employee prevails it will be because a
(A) landowner is strictly liable for injuries occurring on his property.
(B) landowner’s duty of care to third parties cannot be shifted to a tenant by the tenns of a lease.
(C) tenant cannot waive the implied warranty of commercial habitability.
(D) covenant to indemnify by a tenant in favor of a land owner is against public policy.
- (B) At common law, a landlord generally does not impliedly warrant that the leased premises are suitable for any particular purpose, and he is not liable for a dangerous condition existing on the leased premises. Normally, the doctrine of caveat emptor prevails, but there are two notable exceptions. First, Smith and Boyer point out that “a landlord may be liable in tort to the tenant, his guests, licensees and invitees, if at the commencement of the lease there is a dangerous condition that the landlord knows or should know about and the discovery of which would not likely occur by the tenant exercising due care.” Property, pg. 138. Based on this exception, choice (B) is correct because the dangerous condition existed at the commencement of the leasehold period. (Note: The facts indicate that the employee was injured on the first day the tenant took possession of the warehouse.) The second exception is that a landlord in the lease of a completely furnished dwelling for a short period of time impliedly warrants the fitness of the premises and the furnishings. Choice (A) is incorrect. If the landowner is liable, it is based on her/his failure to warn of a dangerous condition that the landlord knew about or should have known about. The landowner’s liability arises from her/his negligence and any claim the employee brings would be based in negligence, not strict liability. Choice (C) is incorrect. This argument, though it sounds similar to choice (B), does not address the fact that the landlord did in fact owe a duty to third parties. This choice generally states that the tenant cannot waive the warranty, but makes no mention of any negligence by the landlord. Because choice (B) does at least mention that the landlord owed a duty of care to third parties, it states a better reason why the landlord is at fault. Choice (C) makes less mention of fault and is a weaker choice than choice (B). Choice (D) is incorrect. This is a weak argument because it gets away from the best theory of liability for the employee. The employee would sue the landlord for the landlord’s own negligence in breaching the duty of care. Though it is not clear that a court would necessarily hold that this type of indemnification provision is against public policy, it is not necessary to engage in this inquiry because this choice does little to explain that the landlord owed a duty to someone in the employee’s position. This choice does nothing to show that the landlord was at all negligent in failing to disclose the defects in the property. Because choice (B) at least mentions that the landlord owed a duty of care to third parties, that choice would be a better argument because it addresses the argument that the landlord was negligent.
- A famous environmentalist was the owner of a large tract of virgin forest. The environmentalist conveyed the tract “to the people of the state in fee simple; provided, however, that if any portion of said tract shall ever be used or developed for any commercial purpose or otherwise converted from its natural state (with exception for recreational, scientific, or educational purposes), then the grantor or his successors in interest may re-enter as of the grantor’s former estate.”
The interest of the state in the tract would be best described as a (an)
(A) fee simple determinable.
(B) fee simple subject to condition subsequent.
(C) easement appurtenant.
(D) determinable fee subject to an executory interest.
- (B) The interest in the tract would be described as a fee simple subject to condition subsequent, which means fee simple subject to being terminated by exercise of the power of termination or right of re-entry for condition broken. The important characteristic that distinguishes this type of estate from a fee simple determinable is that the estate will continue in the grantee, or his successors, unless and until the power of termination is exercised. Although no particular words are essential to create an estate on condition subsequent, the use in the conveyance of the traditional words of condition, “upon condition that,”“provided that,” or “but if,” coupled with a provision for re-entry by the transferor or the occurrence of the stated event will normally be construed to manifest an intention to create an estate on condition subsequent. Choice (A) is incorrect. A fee simple determinable is a fee that automatically expires by force of the special limitation contained in the instrument creating the estate and the fee terminates when the stated contingency occurs. By contrast, the fee simple subject to condition subsequent continues despite the breach of the specified condition until it is divested or cut short by the exercise by the grantor of his power to terminate. Although no particular words are essential to create an estate on condition subsequent, the use in the conveyance of the traditional words of condition, “upon condition that,”“provided that,” and “but if,” coupled with a provision for re-entry bythe grantor will be construed to create an estate on condition subsequent. Choice (C) is incorrect. An easement is the right of one person to go onto the land in possession of another and make a limited use thereof. An easement is appurtenant when it is attached to a piece of land and benefits the owner of such land in his use and enjoyment thereof. Every easement appurtenant requires two pieces of land that are owned by two different persons. The property interest given to the state was a fee simple subject to condition subsequent, which is an actual ownership interest in the land, not merely the right to use the land. Thus, the state was not given an easement. Choice (D) is incorrect. A determinable fee subject to an executory interest is a fee simple estate, where ownership is to pass from the grantee to one other than the grantor upon the happening (or non-occurrence) of a specified event. Because the ownership interest is to pass to the grantor or his successors, this is not an executory interest and this answer choice is incorrect.
- A resident of a state owned a large tract of virgin forest. The resident conveyed this tract “to the people of the state in fee simple so long as said tract shall remain in its natural state (with exception for recreational, scientific, or educational purposes).”
Under state law, conveyances of real property to “the people” of the state are deemed to vest title in the state regarded as a legal entity, and custody over such property resides in an appointed official known as the director of environmental resources. The director granted a logging company the privilege to cut timber on a remote portion of the tract, together with incidental easements of way.
The right of the logging company to cut and sell the timber that was to be cleared from the tract would be an example of a (an)
(A) license.
(B) easement appurtenant.
(C) easement in gross.
(D) profit-a-prendre.
- (D) A profit-a-pendre is similar to an easement in that it is a non-possessory interest. The holder of the profit is entitled to enter upon the servient tenement and take the substance of the land (e.g., minerals, trees, oil, or game) subject to the priviLege. In this regard, a profit, like an easement, may be appurtenant or in gross. Choice (A) is incorrect. A license simply permits one person to come onto the land in the possession of another without being a trespasser. Whereas a profit is an interest in land, a License is not. This is not the best description of the logging company’s rights because the logging company was given more than mere permission to come onto the land. The logging company was aLLowed to remove timber from the property, so their interest is best described as a profit. Choice (B) is incorrect. An easement is the right of one person to go onto the land in possession of another and make a limited use thereof. An easement is appurtenant when it is attached to a piece of rand and benefits the owner of such land in his use and enjoyment thereof. Every easement appurtenant requires two pieces of land that are owned by two different persons. The logging company’s use of the tract does not benefit them as adjacent landowners; rather, their benefit comes from the timber they are allowed to remove. Therefore, it is incorrect to term their interest as an easement appurtenant. Choice (C) is incorrect. An easement is in gross when it is intended to benefit the holder of the easement personally, rather than in connection with any Land the holder owns. In other words, every easement in gross requires only one piece of land (i.e., the servient tenement) that is owned by a person other than the owner of the easement in gross. Because there is no dominant tenement and inasmuch as the logging company has the right to remove timber from the property, a profit is a more accurate description of what the logging company received.
- A resident of a state was the owner of a large tract of virgin forest. The resident conveyed the tract “to the people of the state in fee simple; provided, however, that if any portion of said tract shall ever be used or developed for any commercial purpose or otherwise converted from its natural state (with exception for recreational, scientific, or educational purposes), then the grantor or his successors in interest may reenter as of the grantor’s former estate.”
Under state law, conveyances of real property to “the people” of the state are deemed to vest title in the state regarded as a legal entity, and custody over such property resides in an appointed official known as the director of environmental resources. The director granted a logging company the privilege to cut timber on a remote portion of the tract, together with incidental easements of way. The section that the logging company was to clear would be used for the development of a state recreational area.
After the privilege was granted, the logging company proceeded to clear the timber, which he later sold for $10,000. When the resident learned that the logging company was cutting the timber and selling it, he notified the director to desist further exploitation of the land.
In an ejectment action instituted by the resident against the state to recover the tract, what result would be expected?
(A) The resident would succeed, because the logging company’s selling of the timber would entitle the resident to re-enter and terminate the grant to the state.
(B) The resident would succeed, because the logging company’s selling of the timber would constitute an automatic termination of the state’s fee interest.
(C) The resident would not succeed, because the director approved the clearing of the timber in order to develop the area for recreational purposes.
(D) The resident would not succeed, because the reservation of the resident’s right to terminate would be violative of the rule against perpetuities.
- (C) The resident will not succeed because the director’s objective in the clearing of the timber was for the purpose of developing a recreational area. Although the logging company was not privileged (under the terms of the resident’s deed over to the state) to use the tract for his own commercial advantage, the court will look to the ultimate purpose for which the land was to be used (i.e., recreational park). Thus, the court will conclude that the resident’s original intention and expectation in conveying the tract to the state was not materially or adversely affected. Choice (A) is incorrect. The resident will not succeed because the director’s objective in the clearing of the timber was for the purpose of developing a recreational area. Although the logging company was not privileged (under the terms of the resident’s deed over to the state) to use the tract for his own commercial advantage, the court will look to the ultimate purpose for which the land was to be used (i.e., recreational park). Thus, the court will conclude that the resident’s original intention and expectation in conveying the tract to the state was not materially or adversely affected. For this reason, the resident will not be able to re-enter and terminate the grant to the state. Choice (B) is incorrect. Assuming that the state did violate the condition, that behaviorwould not automaticaLlyterminate the state’s fee interest because the resident granted the state a fee simple subject to condition subsequent. The important characteristic that distinguishes this type of estate from a fee simple determinable is that the estate will continue in the grantee, or his successors, unless and untiL the power of termination is exercised. Because this choice falsely suggests that the estate would automatically be terminated without the resident having to exercise any rights, it is incorrect. This choice is also incorrect because the resident would not prevail because the director’s objective in the clearing of the timber was for the purpose of developing a recreational area. Choice (D) is incorrect. The resident’s right of re-entry is not the type of future interest that requires an application of the Rule against Perpetuities. The Rule against Perpetuities applies to the following future interests: vested remainder subject to open, contingent remainder, and executory interest.
- A farmer was the fee-simple owner of a tract of land in
the country. The farmer moved to the city in 1986. A
man took possession of the land in 1987 as an adverse
possessor and remained on the land from 1987 to 1997;
he then left the property to care for his sick mother for
one year, returning the following year, 1999.
Assume that the statutory period for adverse possession in this state is 20 years.
In 2010, the farmer wanted to move back to the tract because he desired to return to the country. Finding the man in possession, the farmer initiated a suit in ejectment against the man. The most probable judicial determination would be
(A) the farmer would lose, because the man’s possession was open, hostile, and continuous.
(B) the farmer would lose, because the man had fulfilled the required statutory period to become an adverse possessor.
(C) the farmer would win, because the man’s possession was not continuous.
(D) the farmer would win, because he is the titleholder of record.
- (C) The farmer would win because the man’s possession was not continuous. For one to hold property adversely, his possession must be (1) actual and exclusive; (2) open and notorious; (3) continuous and peaceable (meaning without any interruption); and (4) hostile and adverse. Because the man’s possession was interrupted, he didn’t fulfill the statutory requirement of continuous possession for 20 years. Because the man never fulfilled an unbroken 20-year period, the man cannot claim ownership through adverse possession. Choice (A) is incorrect. The man’s possession was not continuous for the 20-year statutory period because he left for one year to care for his sick mother. When the man returned in 1999, that reset the statutory period and the man needed to fulfill a new 20-year period. Choice (B) is incorrect. The man did not fulfill the required statutory period to become an adverse possessor because he left for one year to care for his sick mother. When the man returned in 1999, that reset the statutory period and the man needed to fulfihla new 20-year period. Choice (D) is incorrect. The farmer does not win simply because he is the title holder of record because if the man had been in continuous possession, then the man would be allowed to claim the tract. Rather, the better reason why the farmer wins is because the man cannot fulfill all of the requirements of adverse possession.
- A woman was the owner of a farm in fee simple. The
woman left the farm in 1986 to move to the city. A
man took possession of the land in 1987 as an adverse
possessor and remained on the land from 1987 to 1997;
he then left the property to care for his sick mother for
one year, returning the following year, 1999.
The man’s nephew agreed to live on the farm and take care of it during his uncle’s absence. When the man returned from his mother’s home in 1999, he continued in possession until 2010 when the woman
brought an ejectment action.
Assume that the statutory period for adverse possession in this state is 20 years. The court would most likely hold that
(A) the woman was the rightful owner and thus entitled to possess the farm.
(B) the woman was entitled to possession, because the man’s intermittent possession was insufficient to make him an adverse possessor.
(C) the man was entitled to possession, because his possession was open, notorious, continuous, and hostile as required for the statutory period.
(D) the man was entitled to possession, because of the privity between the man and his nephew, which aided him in fulfilling the required statutory period.
- (D) Choice (D) is correct because the period of adverse possession of one possessor can be tacked on to the period of adverse possession of another possessor when there is privity (e.g., blood or contract) between the two. Choice (A) is incorrect. Because the nephew’s time spent can be tacked on to both segments of the man’s occupation, the man will be able to show that his possession was continuous forthe requisite 20 years. Thus, the woman wiLl not be declared the rightful owner. Choice (B) is incorrect. Although the man’s possession was intermittent and broken, the man will be allowed to tack his two segments of possession onto the nephew’s possession and create a continuous period. Choice (C) is incorrect. The man is, in fact, entitled to possession, but only because he was allowed to tack his time onto the nephew’s stay. Because this choice fails to take this factor into account, it is not as complete as choice (D), which does mention that it is the man’s tacking on that is the reason he will prevail.
- A rancher was the owner of a parcel of land in fee simple. The rancher left the parcel in 1977 to move to the city. A man took possession of the land in 1978 as an adverse possessor and remained on the land from 1978 to 1988; he then left the property to care for his sick mother for one year, returning the following year, 1990. Meanwhile, during the man’s possession, the rancher devised the parcel to his son in 1994. The man remained on the property until
2010.
Assume that the statutory period for adverse possession in this state is 20 years.
Upon the rancher’s death in 2010, his son, as his sole heir, brings an ejectment suit to recover possession of the property from the man. In his action, the son will most probably
(A) lose, because the man has acquired title by adverse possession.
(B) win, because the statutory period for adverse possession does not run against a remainder- man until his interest becomes possessory.
(C) win, because the man left the property to take care of his mother for one year.
(D) win, because his title was perfected at the time of conveyance.
- (A) Since the man was in continuous possession of the parcel from 1990 to 2010, he would acquire title by adverse possession. As a result, choice (A) is correct. Choice (B) is incorrect because the facts do not disclose that the son was a remainderman or had a remainder interest in the parcel. The rancher simply devised the property to his son in 1994. As a consequence, the son did not have a remainder interest but a present possessory interest as of 1994. Choice (C) is incorrect. Certainly, the man left the property to take care of his mother and this created a gap in the continuous nature of the man’s possession. But when the man returned in 1990, he fulfilled a full statutory period of 20 years because the son is bringing suit in 2010. Choice (D) is incorrect. The son’s title was not perfected in 1994 simply because it was conveyed to him. In fact, there was no attempt to perfect title until the son sued for ejectment and tried to quiet title. That attempt to perfect title did not occur until 2010 and by that point, the man had fuLfilled the statutory period for adverse possession.
- A homeowner died in 1985. His will devised his estate in a southern state to his uncle and his best friend “to share and share alike as tenants in common.”
At the time of the homeowner’s death, the uncle lived in a different part of the southern state (in which the estate was located), while the best friend resided in a northern state. After the homeowner’s funeral, the uncle returned to his own residence, but the best friend decided to occupy the estate. He put his name on the mailbox and has paid the taxes and maintenance expenses. To earn extra money, the best friend rented a small house on the property to a teacher and received a monthly rental payment from her. The best friend also grew fruits on the property and sold them at a stand on Fridays. The uncle has been generally aware of this, but because he cared little about the estate, the uncle has never pressed the best friend about the property. Since 1985 the uncle has not paid any rent or other compensation to the best friend, nor has the best friend requested such payment.
In January 2010, a series of disputes arose between the uncle and the best friend for the first time concerning their respective rights to the estate. The state in which the property is located recognizes the usual common law types of cotenancies and follows majority rules on rents and profits. There is no applicable legislation on the subject.
The uncle brings an appropriate action for a portion of the proceeds that the best friend received from his fruit stand and a portion of the rent that the teacher paid. If the best friend contests the apportionment of the monies he received, judgment should be for whom?
(A) As a cotenant in possession, the best friend retains the profits from his crops and the rents paid by the teacher.
(B) As a cotenant in possession, the best friend retains the profits from his crops, and the uncle is entitled to a share of the rent paid by the teacher.
(C) The uncle is entitled to a share of the rent that the teacher paid and the profits from the best friend’s crops.
(D) The uncle is entitled to no share of any of the monies raised because the uncle’s lack of contact with the best friend will be deemed a waiver.
- (B) In a majority of states, a tenant in possession has the right to retain profits gained by his use of the property. However, a co-tenant out of possession does have a right to share in rents and profits received from third parties. The best friend will be allowed to keep all of the money he earned from his fruit stand and need not share those profits gained by his own use of the property. However, the uncle wiLl be entitled to a share of the rents paid by the teacher. Choice (A) is incorrect. Even though the best friend was the co-tenant in possession, he is not entitled to keep all of the rent paid by the teacher. The best friend must share a portion of the rent with the uncle. Choice (C) is incorrect. In a majority of states, a tenant in possession has the right to retain profits gained by his use of the property. However, a co-tenant out of possession does have a right to share in rents and profits received from third parties. The best friend will be allowed to keep all of the money he earned from his fruit stand and need not share those profits gained by his own use of the property. However, the uncle will be entitled to a share of the rents paid by the teacher. Choice (D) is incorrect. Even though the best friend was the co-tenant in possession, he is not entitled to keep all of the rent paid by the teacher. The best friend must share a portion of the rent with the uncle.
- A man died in 1985. His will devised his estate in a Midwestern state to his son and daughter “to share and share alike as tenants in common.”
At the time of their father’s death, the daughter lived in a different part of the Midwestern state (in which the estate was located) while the son resided in a state in the Southwest. After the funeral, the daughter returned to her own residence, but the son decided to occupy the estate. He put his name on the mailbox and has paid the taxes and maintenance expenses. The daughter has been generally aware of this, but because she cared little about the estate, the daughter has never pressed her brother about the property. Since 1985, the daughter has not paid any rent or other compensation to her brother, nor has he requested such payment.
In January 2010, a series of disputes arose between the son and daughter for the first time concerning their respective rights to the estate. The state in which the property is located recognizes the usual common law types of cotenancies and there is no applicable legislation on the subject. Now, the son claims the entire fee simple absolute title to the estate and brings an action against his sister to quiet title in himself.
If the Midwestern states in which the property is located has an ordinary 20-year adverse possession statute, judgment should be for whom?
(A) The daughter, because one cotenant cannot acquire title by adverse possession against another.
(B) The daughter, because there is no evidence that her brother has performed sufficient acts to constitute an ouster.
(C) The son, because the acts of the parties indicate the sister’s intention to renounce her ownership rights to the estate.
(D) The son, because during the past 25 years, the son has exercised the type of occupancy ordinarily considered sufficient to satisfy adverse possession requirements.
- (B) Under the unity of possession, each co-tenant is entitled to possess and enjoy the whole of the property subject to the equal right of his (or her) co-tenant. If one tenant wrongfully excludes another co-tenant from possession of the whole or any part of the whole of the property, such conduct amounts to an ouster. Here, choice (B) is correct because there is no evidence that the son was wrongfully excluding his sister from the estate. Choice (A) is incorrect. (A) is wrong because Burby points out that “possession may become adverse if the co-tenants are excluded from possession (i.e., ouster) or if the co-tenants are specifically notified of the adverse claim or can be charged with notice because of the acts of the possessor.” Real Property, pg. 232. Because one co-tenant can acquire title by adverse possession against another if there are acts sufficient to constitute ouster, this choice is incorrect. Choice (C) is incorrect. Ouster occurs when one co-tenant manages to wrongfully exclude his co-tenants from possession of the property. If there were a proper ouster, it is the actions of the son (as the co-tenant in possession) that would be relevant. The daughter’s actions and behavior alone cannot constitute an ouster because an ouster will be determined by whether or not the son acted to wrongfully exclude his sister. Choice (D) is incorrect. Even if this were true and the son behaved in such a way as to satisfy the requirements of adverse possession, there must still be acts sufficient to constitute an ouster of the daughter before the son can adversely possess the entire estate. This answer choice fails to take into account the need for ouster, thus making this choice incomplete.
- A rancher is currently the owner of record of a parcel of land designated as “A.” It is the higher of two neighboring properties on a mountain. A farmer is the owner of parcel “B,” which is situated lower than “A” on the mountainside.
In 2000, the rancher, who owned both parcels of land, constructed a private drainage system. This system consisted of an underground pipeline that extended across both parcels of land.
Sewage from parcel “A” drained through the system to a municipal sewer, which was located at the bottom of the mountain.
In 2005, the rancher sold parcel “B” to the farmer. The deed to the farmer did not contain any mention of the rancher’s drainage system. Located on each of the parcels are large ranch-style homes occupied by the respective owners.
Lately, the sewage drain has begun to emit an unpleasant odor across parcel “B.” As a result, the farmer sued the rancher in trespass praying for damages caused by the drainage system.
Judgment should be for
(A) the rancher, because the deed to the farmer did not contain any covenants restricting the rancher’s use of the land.
(B) the rancher, because the farmer’s proper cause of action would be for nuisance, rather than trespass.
(C) the farmer, because the drainage system was the proximate cause of the plaintiff’s damages as a matter of law.
(D) the farmer, because the rancher did not have the right to improve his own land in a manner that caused harm to another.
- (B) The farmer’s proper cause of action should be based on nuisance, rather than trespass. The basic distinction that is now recognized is that trespass is an invasion of the plaintiff’s interest in the exclusive possession of his land, whereas nuisance is an interference with his use and enjoyment of it. In all likelihood, the rancher has an implied easement for his sewer pipe from parcel “A” across parcel “B.” It is generally agreed that in cases involving an implied reservation (or quasi-easement) in favor of the grantor, there must be reasonable necessity forthe existence of the implied easement. Choice (A) is incorrect. Just because the deed contained no covenants restricting the rancher’s use, that doesn’t allow the rancher to use the easement in such a way as to interfere with the use and enjoyment of the farmer’s property. The holder of an easement may make reasonable use of the servient tenement so long as her conduct does not unreasonably interfere with the use of the servient tenement owner. Because the type of harm that the farmer is suffering is more appropriately redressed under a nuisance action, Choice (B) is better. Because choice (A) does not address the fact that the farmer is bringing an improper cause of action, it is not as complete as choice (B). Choice (C) is incorrect. The farmer’s proper cause of action would be a suit in nuisance (due to the odors), not negligence. For this reason, proximate cause will not need to be part of the analysis. Choice (D) is incorrect. The farmer’s proper cause of action would be a suit in nuisance. Because this answer choice names the farmer as the prevailing party in a trespass suit against the rancher, it is an incorrect choice.
- A man is the owner of record of a parcel of land designated as “A.” It is the highest of three neighboring properties on a mountain. The man’s friend is the owner of parcel “B,” which is situated lower than “A” on the mountainside. The man’s cousin owns parcel “C,” which lies below parcels “A” and “B” on the mountain slope.
In 2000, the man who originally owned all three parcels of land constructed a private drainage system. This system consisted of an underground pipeline that extended across all three parcels of land.
Sewage from parcel “A” drained through the system to a municipal sewer, which was located at the bottom of the mountain.
In 2005, the man sold parcel “B” to his friend. The following year, the cousin purchased parcel “C” from the man. The deeds to the friend and the cousin did not contain any mention of the man’s drainage system. Located on each of the parcels are large ranch-style homes occupied by the respective owners.
From the standpoint of the friend and the cousin, the
drainage system would most likely be defined as a
(an)
(A) easement appurtenant.
(B) easement by implication.
(C) prescriptive easement.
(D) express easement.
- (B) An easement may arise by implication if the existence of the easement is strictly (or reasonably) necessary forthe beneficial use of the land. The creation of such an easement is based upon the presumed intent of the grantor and grantee. If the claim is made that the easement arose out of a prior conveyance, there must be proof of the fact that at one time both the dominant and servient estates were under one ownership. Technically, choice (A) is correct, but choice (B) is preferred because the present example is an illustration of an implied easement appurtenant. So choice (A) is incorrect. The easement is appurtenant in that it is attached to a piece of land and benefits the owner of such land in his use and enjoyment thereof. However, this choice still does not explain how the easement was created. Because choice (B) better explains how the easement was created, it is a preferred choice. Choice (C) is incorrect. With respect to (C), an easement (or profit) may also be acquired by prescription in a manner similar to that by which ownership of apossessory estate may be acquired by adverse possession. However, because there are facts that support an easement created by implication (common ownership of both estates and reasonable necessity for creation of the easement), it is a stronger argument for the man to make than to argue that this is an easement created by prescription. Choice (D) is incorrect. This is not an express easement because the facts state that neither the friend’s nor the cousin’s deed made any reference to the man’s pre-existing drainage system.
- A woman is the owner of record of a parcel of land designated as “A.” It is the highest of three neighboring properties on a mountain. The woman’s sister owns parcel “C,” which lies the lowest on the mountain. Parcel “B” lies between Parcels “A” and
In 2000, the woman who originally owned all three parcels of land constructed a private drainage system. This system consisted of an underground pipeline that extended across all three parcels of land.
Sewage from parcel “A” drained through the system to a municipal sewer, which was located at the bottom of the mountain.
In 2005, the sister purchased parcel “C” from the woman. The deed to the sister did not contain any mention of the woman’s drainage system. Located on each of the parcels are large ranch-style homes occupied by the respective owners. A portion of the sister’s unimproved land began to subside over the drainage system. This was because of an especially pliable soil condition of which the woman was unaware prior to construction of the drainage system.
In an action brought by the sister against the woman to recover property damage for the subsidence of her land, judgment should be for whom?
(A) The sister, because she is entitled to lateral support for her land in its natural condition.
(B) The sister, because she is entitled to subjacent support for her land in its natural condition.
(C) The sister, because she is entitled to support for her land in its improved condition.
(D) The woman, unless she sister proves that the woman was negligent in constructing the drainage system.
- (B) The right to subjacent support means support from underneath the surface of the land as distinguished from the sides. Unlike with lateral support, a landowner has an absolute right to subjacent support for land in its natural state and for any structures on the land. Choice (A) is incorrect. The sister would not be bringing an action for loss of lateral support in this instance. Because the sister suffered property damage for the subsidence of her land, she will be suing for a loss of subjacent support. Choice (C) is incorrect. The sister only suffered damage to an unimproved portion of her property when the subsidence began. Because there are no facts to suggest that any structure of the ranch-style home was damaged, this is not the best choice. Choice CD) is incorrect. The sister does not need to prove negligence on the part of the woman in order to recover for the property damage due to a loss of subjacent support.
- A developer built a mini-mall in the city. In order to finance the construction of the mall, the developer obtained a $500,000 commercial loan from a bank. The loan was secured by a mortgage that contained an after-acquired property clause that provided:
“(The developer) hereby covenants that the terms of this mortgage shall apply to all after-acquired property which said mortgagor shall hereafter acquire; and
Mortgagee-bank’s security interest in such after- acquired property shall take priority over all other parties claiming an interest in such property.”
The bank duly recorded this mortgage on August 15, 2007. On September 6, 2008, the developer purchased a four-story office building in a nearby city. The building was in need of much renovation. To finance the improvements, the developer borrowed $250,000 from his friend. As consideration for the loan, the developer executed a promissory note for $250,000, payable to the friend in 10 years and secured by a mortgage on the office building. This mortgage was dated September 9, 2008 and recorded the following day.
During the early part of 2009, the developer experienced serious financial setbacks and failed to make the monthly mortgage payments to the bank and the friend. After the developer went into default on both loans, the bank instituted a foreclosure action on both the mini-mall and the office building. The friend intervened, claiming that his security interest in the office building took priority over the after-acquired rights claimed by the bank.
This jurisdiction maintains a grantor—grantee index and has the following recording statute in effect:
“An unrecorded mortgage, deed, or other instrument is invalid as against a subsequent mortgagee or bona fide purchaser for value without notice.”
In determining which security interest in the office building should be given priority, the court should hold in favor of
(A) the bank, because its mortgage was recorded prior in time.
(B) the bank, because the after-acquired property clause in its mortgage is enforceable and takes priority over the friend’s subsequent security interest.
(C) the friend, because the bank’s prior recorded mortgage is outside the friend’s chain of title.
(D) the friend, because an after-acquired property clause is void against public policy and cannot encumber real estate subsequently acquired by the mortgagor.
- (B) According to Osborne, mortgages on after-acquired property are valid as between the parties, “but to be effectual against third persons, it must point out the subject matter so that such persons by inquiry notice may be able to identify the property intended to be covered.” In this regard, Osborne further notes that “a clause covering all after-acquired property is good” and is sufficient to adequately identify the property between the mortgagor-mortgagee and to put third parties on notice. Mortgages, p. 69. In this question, the facts state that the developer’s mortgage with the bank covered “all after-acquired property.” Therefore, the description is sufficiently definite to cover the office building subsequently acquired by the developer. Choice (A) is incorrect. Although this is true that the bank recorded its mortgage prior in time, this choice does not explain why the bank has an interest in property that was mortgaged to another party. Choice (B) gives a more specific reason why the bank prevails because (B) explains that it is comprehensive nature of the after-acqufred property clause that gives the bank a security interest in this second property. By recording this interest prior in time, this gave constructive notice to the friend that the bank would have an interest in all property acquired later in time by the developer. Because this choice only states a fact without explaining why that is legally significant, it is incomplete. Choice (C) is incorrect. The friend could look in the Grantor Index under the developer’s name to see what the developer did as a grantor. The friend would find that the developer had given a mortgage to the bank that was recorded in August 2007. In that mortgage was the after-acquired property clause. The mortgage would not be outside the friend’s chain of title. Choice (D) is incorrect. In this question, the facts state that the developer’s mortgage with the bank covered “all after-acquired property.” Therefore, the description is sufficiently definite to cover the office building subsequently acquired by the developer. Because of this, the clause is valid and will not be declared void as against public policy.
- A senator, had been a state senator for three years. During that period of time, it had been his regular practice to stay at a hotel in the state’s capital while the legislature was in session (namely from September through March). As a general rule, the senator stayed at the hotel each and every night during that seven-month period. When the senator checked into the hotel on September 1, 2007 for the 2007—2008 term, the hotel manager quoted the senator his usual senatorial discount rate of $50 per night. Although nothing was said regarding payment terms, the senator customarily paid the bill in weekly installments.
During the months of September and October, he paid the hotel at the end of each week $350 plus tax and other accumulated expenses. On the last day of October, the senator made his usual weekly payment covering the rental period from October 25 to October 31. Then, on the morning of November 4 (before checkout time for that day), the senator decided to suddenly move out of the hotel and register at a new motel located two blocks away. He notified the manager at the hotel of his intent to check out and tendered a check for $150 plus tax and other incidental charges incurred from November 1 through November 3. When the manager noticed that the senator was paying only for three days, she remarked, “You still owe us $200 plus tax for the rest of the week and $350 plus taxes for an additional week because you didn’t give us a week’s notice that you planned to vacate.” The senator refused to pay the additional charges.
If the hotel brings suit against the senator to recover for the unpaid balance, how many additional days lodging will he be required to pay for?
(A) None.
(B) Four.
(C) Seven.
(D) Eleven.
- (A) Every leasehold includes the following elements: (1) an estate in the tenant; (2) a reversion in the landlord; (3) exclusive possession and control of the land in the tenant; and (4) generally, a contract between the parties. The first three elements constitute privity of estate, and the fourth constitutes privity of contract. Whereas in a lease exclusive possession must reside in the tenant, in a license the possession remains in the licensor and the licensee has a mere privilege of being on the land without being treated as a trespasser. Smith and Boyer, Real Property, p. 48. Ordinarily, a lodging contract does not create a landlord—tenant relationship but only that of licensor and licensee, whereby the occupant contracts for the use of a room and facilities without a possessory interest in the land. Exclusive possession and control remains in the landowner. The senator, as a hotel guest, does not have exclusive possession and control of his hotel room. He is a licensee, rather than a tenant. Furthermore, the facts do not strongly support a periodic tenancy since his oral agreement with the hotel was not referred to as a “lease,” nor was the weekly payment referred to as “rent.” Therefore, no seven-day prior notice will be required. In the absence of any leasehold, the senator will owe no additional balance beyond the day he checks out. Choice (A) is correct. Choice (B) is incorrect. This is the balance of the week that the senator failed to complete. He was not required to lease in whole weeks since the facts do not strongly support a periodic tenancy because his oral agreement with the hotel was not referred to as a “lease,” nor was the weekly payment referred to as “rent.” Because no periodic tenancy had been created, there was no reason why the senator needed to pay for an entire week’s worth of days. He could leave the hotel room in the middle of a given week and only pay for those days that he had occupied the room. Choice (C) is incorrect. If a periodic tenancy had been created, then the senator would likely have to pay for eleven days—the four remaining on this week plus the next seven days because he would have to give one full period’s notice before moving out. Seven days would not make sense in this situation because the senator would not be required to pay only one more week from the date in question; he would have to pay for eleven days. Choice (D) is incorrect. The facts do not strongly support a periodic tenancy because his oral agreement with the hotel was not referred to as a “lease,” nor was the weekly payment referred to as “rent.” Because no periodic tenancy had been created, there was no reason why the senator needed to pay for an entire week’s worth of days. He could leave the hotel room in the middle of a given week and only pay for those days that he had occupied the room. Because of this, the senator was not required to give one full period’s notice before moving out and not required to pay for that extra time that he would be staying in the room.
- A woman is the fee simple owner of a 1,000-acre tract of land. Half of the tract is swampland totally unfit for cultivation, though it is heavily covered with valuable hardwood timber. The other half is also covered with valuable timber, but it would be land fit for the raising of crops if it were cleared of the timber. The latter section of land is more valuable for cultivation than it is for the growing of timber and has clearings where someone could build a home and live.
The woman conveyed the tract to her brother for life. At the time of the conveyance, the swampland had never been used for the production of timber. The brother took possession and cleared 40 acres of the timber on the section that was suitable for cultivation. In addition, the brother cut 60 acres of timber in the swampland, thus becoming the first person to exploit this area. The brother then proceeded to clear some timber on the tract and used it in repairing fences, buildings, equipment, and the like on the property.
In an action by the woman to permanently enjoin the brother from cutting any more timber on the swampland section of the tract and to account for profits received in the sale of the timber, the woman will most likely
(A) succeed, because a life tenant must account for permissive waste to the reversioner or remainderman.
(B) succeed, because a life tenant may not exploit natural resources where no such prior use had been made.
(C) not succeed, because a life tenant is not liable for permissive waste.
(D) not succeed, because a life tenant has a right to make reasonable use of the land.
- (B) The swampLand is not usable for the growing of crops, nor can it be made to be. It seems to be capable of growing trees only. This swampland, as far as the facts disclose, has never been used (and at the time of the taking effect of the brother’s life estate) was not being used for the production of lumber. The brother is the first to cut timber on the property. It is well settled that a tenant for life (or for years) has a right to continue the operation of old mines on the leased premises for the reason that such is the use to which the land has been put or is being put at the time of the beginning of the tenant’s estate. The lease implies that such use may be continued. On the other hand, such tenants have no right to open new mines and appropriate the minerals. Likewise, the timber should not be available for consumption and destruction by a tenant of a limited estate as against the reversioner or remainderman who owns the inheritance. Therefore, the brother should be permanently enjoined from cutting any more trees on the swampland, and should account for the proceeds from the sale of the timber that was cut on the swampland. Choice (A) is incorrect. Permissive waste consists of injury to the premises or land caused by the tenant’s failure to act when it is her/his duty to act (e.g., there is a hole in the roof of a leased dwelling house, which the tenant fails to repair, thus causing rain to leak and damaging the hardwood floors of the house). This is not the type of waste that the brother engaged in because he did not allow the premises to fall into a state of disrepair due to his neglect. If anything, he did some affirmative act to the land. Choice (C) is incorrect. A court can grant an injunction against the life tenant to enjoin any further waste on the property. Moreover, the court can also order an accounting for waste already committed by the life tenant. Because this answer choice falsely suggests that a Life tenant is not liabLe for permissive waste, it is incorrect. Choice (D) is incorrect. Certainly, a life tenant has the right to make reasonable use of the Land. What the life tenant does not have the right to do is to use (or neglect) the land in such a way as to impair the interests of the holder of any reversion or remainder interest in the land.
- A grantor is the fee simple owner of a 1,000-acre tract of land. Half of the tract is swampland totally unfit for cultivation, though it is heavily covered with valuable hardwood timber. The other half is also covered with valuable timber, but it is land that would be fit for the raising of crops if it were cleared of the timber. The latter section of land is more valuable for cultivation than it is for the growing of timber.
The grantor conveyed his tract to his brother for life. At the time of the conveyance, the swampland had never been used for the production of timber. The brother took possession and cleared 40 acres of the timber on the section that was suitable for cultivation. In addition, the brother cut 60 acres of timber in the swampland, thus becoming the first person to exploit this area.
The brother’s cutting of the timber on the section of tract suitable for cultivation would be an example of
(A) permissive waste.
(B) ameliorative waste.
(C) equitable waste.
(D) unreasonable exploitation.
- (B) Ameliorative waste is a change in the physical characteristics of the occupied premises by an unauthorized act of the tenant that increases the value of the property.
The law takes account of this by awarding to the tenant the timber that is removed for such purpose. Thus, the tenant for life (or for years) has a right to use land as good husbandry dictates. As such, this usually incLudes the right to transform timberland into arable land for the plow, provided of course the soil can be made suitable for such purpose. Choice (A) is incorrect. Permissive waste consists of injury to the premises or land caused by the tenant’s failure to act when it is her/his duty to act (e.g., there is a hole in the roof of a leased dwelling house, which the tenant falls to repair, thus causing rain to Leak and damaging the hardwood floors of the house). This is not the type of waste that the brother engaged in because the brother did not allow the premises to fall into a state of disrepair due to his neglect. If anything, he did an affirmative act to the land. Choice (C) is incorrect. Equitable waste consists of injury to the reversionary interest in land, which is inconsistent with good husbandry. It is recognized only by the equity courts and does not constitute legal waste. It usually arises when the expression “without impeachment of waste” appears in the grant, which means that the life tenant or tenant for years may use the premises as a fee simple owner might use the Land. Choice (D) is incorrect. Although the brother’s behavior might be construed as an unreasonabLe exploitation of the timber, his conduct is more appropriately termed ameliorating waste. Because choice (B) is a more specific answer, choice (D) is incorrect.
- A woman and her husband purchased a home in
1982 and recorded the deed as tenants in common. After 10 years of marriage, the couple began experiencing marital difficulties. In 1992, they separated and the husband moved to another state where his family lived. The woman continued to reside in the dwelling.
In 1994, the husband quitclaimed his interest in the home to his son, who promptly recorded his deed. In 1995, the couple divorced. In 1996, the woman changed the locks to the home, preventing the son from entering the property, and the woman refused the son’s repeated demands to allow him access to the home and property. From 1995 to 2005, the woman paid all the taxes and insurance on the property. She also was responsible for making whatever repairs were necessary. The period of adverse possession in this jurisdiction is 10 years.
In 2006, the woman filed suit to quiet title to the home claiming absolute fee simple ownership. The son has countersued asserting that he owns an undivided 50 percent interest in the property. If the court rules in favor of the woman, it will be because
(A) the son will be unjustly enriched, inasmuch as the woman has paid all the taxes, insurance, and made necessary repairs.
(B) the son acquired his interest in the property by quitclaim deed.
(C) the deed from the husband to the son was void for lack of consideration.
(D) the woman obtained the son’s interest by adverse possession.
- (D) Choice (D) is correct because possession may become adverse if there is an ouster, in which case, one co-tenant can acquire title against the other. If one tenant wrongfully excludes another co-tenant from possession of the whole or any part of the whole of the property, such conduct amounts to an ouster. By refusing the son’s request to live in the house and by changing the locks, the woman acted in derogation of one of the son’s rights of possession. Choice (A) is incorrect. This choice suggests that the woman would retain an entire fee simple interest in the property for the reason that it would be inequitable to allow the son to have an ownership interest because she paid all the taxes and insurance and made repairs. This would mean that those actions alone would be enough for the woman to retain an entire fee simple interest in the property. Without a proper ouster, the woman cannot own the entire estate just by those actions alone. The woman owns the property because she ousted the son for the entire statutory period. That is why she will prevaiL and not because of her payment of expenses and making repairs. Choice (B) is incorrect. This choice suggests that the woman would prevail simply because the son took via a quitclaim deed. A quitclaim deed will only be reLevant if the son is suing the husband for violation of one of the six deed covenants. However, in the woman’s contest with the son for title to the property, how the son tooktitle is irrelevant to the woman’s success in her quiet title action. Choice (C) is incorrect. Even if the transaction between the husband and the son were somehow invaLid, the fact remains that the husband would then own the other undivided one-half interest in the property. This answer choice still does not explain how the woman acquired title to all of the property.
- An investor and a developer own adjoining lots in the central portion of a city. Each of their lots had an office building on it. The developer decided to raze the existing building on her lot and to erect a building of greater height. The developer has received all governmental approvals required to pursue her project.
There is no applicable statute or ordinance (other than those dealing with various approvals for zoning, building, etc.).
After the developer had torn down the existing building, she proceeded to excavate deeper. The developer used shoring that met all local, state, and federal safety regulations, and the shoring was placed in accordance with those standards.
The investor notified the developer that cracks were developing in the building situated on the investor’s lot. The developer took the view that any subsidence suffered by the investor was due to the weight of the investor’s building and correctly asserted that none would have occurred had the investor’s land been in its natural state. The developer continued to excavate. The building on the investor’s lot did suffer extensive damage, requiring the expenditure of $750,000 to remedy the defects.
Which of the following is the best comment concerning the investor’s action to recover damages from the developer?
(A) The developer is liable, because she removed necessary support for the investor’s lot.
(B) The developer cannot be held liable simply upon proof that support was removed, but may be held liable if negligence is proved.
(C) Once land is improved with a building, the owner cannot invoke the right of lateral support.
(D) The developer’s only obligation was to satisf’ all local, state, and federal safety regulations.
- (B) The right of a landowner to have her land supported lateraLLy by the neighboring land is an absolute right inherent in the land itself. The right of lateral support means the land in its natural condition. The right of lateral support does not include the right to have the additional weight of artificial structures supported. If there is damage to artificial structures, the majority view is recovery is limited to damage to the land in its natural condition and may not include any damage to the artificial structures on the land. However, if there is negligence on the part of the wrongdoer who removes lateral support, then the defendant is liable for all damages that proximately result, including damages to both the land and artificial structures. Choice (A) is incorrect. If there is negligence on the developer’s part during her removal of lateral support, then she would be liable for the damage done to the investor’s building. However, there is never strict liability for damage done to artificial structures. This choice falsely suggests that the developer would be liable just because she removed the necessary support, thus creating strict liability on the developer’s part. Because this is not the test, this choice is incorrect. Choice (C) is incorrect. If there is negligence on the developer’s part during her removal of Lateral support, then she would be liable for the damage done to the investor’s building. Because this choice suggests that the investor’s erecting a building on her property has somehow foreclosed any right she has to lateral support, it is incorrect. Choice (D) is incorrect. Even if the developer were in compliance with all local, state, and federal safety regulations, she still owed a duty to the investor to maintain the investor’s right to lateral support. If there were negligence on the developer’s part, she would be Liable to the investor and it would be no defense that she was in compliance with these regulations.
- A landowner and a developer own adjoining lots in the central portion of the city. Each of their lots had an office building built on it. The developer decided to raze the existing building on her lot and to erect a building of greater height. The developer has received all governmental approvals required to pursue her project.
When the building was completed, the homeowner discovered that the shadow created by the new higher building placed her building in such deep shade that her ability to lease space was diminished and that the rent she could charge and the occupancy rate were substantially lower. Assume that these facts are proved in an appropriate action that the landowner instituted against the developer for all and any relief available.
Which of the following is the most appropriate comment concerning this lawsuit?
(A) The landowner is entitled to a mandatory injunction requiring the developer to restore conditions to those existing with the prior building insofar as the shadow is concerned.
(B) The court should award permanent damages, in lieu of an injunction, equal to the present value of all rents lost and loss on rents for the reasonable life of the building.
(C) The court should award damages for losses suffered to the date of trial and leave open recovery of future damages.
(D) Judgment should be for the developer, because the investor has no cause of action.
- (D) In most jurisdictions, negative easements for light, air, and view have been repudiated, absent an express grant between the parties, because such restrictions would seriously hamper land development. Burby, Real Property, pg. 75. In this question, the developer will prevail because she is not liable for the shadow created by her building. Choice (A) is incorrect. In most jurisdictions, negative easements for light, air, and view have been repudiated. Therefore, the landowner will not be entitled to an injunction against the developer’s construction of her building. Choice (B) is incorrect. In most jurisdictions, negative easements for Light, air, and view have been repudiated. Therefore, the landowner will not be entitled to permanent damages for the developer’s construction of her building. Choice (C) is incorrect. In most jurisdictions, negative easements for light, air, and view have been repudiated. Therefore, the landowner will not be entitled to damages for losses suffered from the developer’s construction of her building.
- In 2001, the owner of a mansion and its surrounding property died. In his will, he devised the property to “my son and his heirs, but if my son dies without issue, to my daughter and her heirs.”
Assume that when the owner died, the daughter was still alive. The daughter did not have any children at the time that the owner died. However, 10 years after her father’s death, the daughter had a child.
After the owner’s death the daughter’s interest in the property may best be described as a
(A) contingent remainder.
(B) springing executory interest.
(C) shifting executory interest.
(D) vested remainder, subject to complete divestiture.
- (C) After the owner’s death, the daughter’s interest in the property could best be described as a shifting executory interest (or executory devise because it was created by will), which divests the preceding estate upon the happening (or non-happening) of a stated event. Executory devises are interests that are identical with springing and shifting uses, except that they are created by will, instead of by deed. Thus, the owner devised a fee simple to his son and his heirs, which was subject to the daughter’s executory interest. If the event happens (the son dies without issue), then the daughter will have a possessory estate in fee. Choice (A) is incorrect. A remainder is a future interest created in a third person that is intended to take after the natural termination of a preceding estate. A remainder is contingent if it is created in favor of an ascertained person, but (1) is subject to a condition precedent, or (2) is created in favor of an unborn person, or (3) is created in favor of an existing but unascertained person. The daughter’s interest is not a remainder because she does not take after the natural termination of the preceding estate; rather, she takes only upon the happening (or non-happening) of a specified event. This is why her interest is best categorized as an executory interest. Choice (B) is incorrect. A springing executory interest is an estate created to begin in futuro and “cuts short” or terminates a reversion held by the grantor. A springing executory interest is one that divests a grantor of the fee interest. Here, the interest was held by the son and not by the grantor. Thus, the executory interest is of the shifting type, not springing. Choice (D) is incorrect. A remainder is vested subject to complete divestment when the remainderman is in existence and ascertained and her interest is not subject to a condition precedent, but her right to possession or enjoyment on the expiration of the prior interests is subject to termination by reason of(1) an executory interest, or (2) a power of appointment, or (3) a right of entry. However, the daughter does not have a remainder because she does not take after the natural termination of the preceding estate; rather, she takes only upon the happening (or non-happening) of a specified event. This is why her interest is best categorized as an executory interest.
- In 1963, Smith, the record owner of Winterthur died and in his will he devised Winterthur to “my son, Samuel, and his heirs, but if Samuel dies without issue to my daughter, Dixie, and her heirs.” Then Samuel deeded to “Bailey and his heirs,” which was executed on July 1, 1973 and recorded on the same date.
In 1974, Bailey died and in his will he devised Winterthur to “my brother Corley for life, then to Corley’s heirs.” In 1983, Corley executed a general warranty deed in which he conveyed “all my right, title and interest in Winterthur to my children, Jesse and James.” James died intestate in 1988, leaving his 14-year-old son, Butch, as his only heir. Corley died in 1993, leaving Butch and Jesse as heirs.
In 1998, Butch and Jesee conveyed Winterthur by a general warranty deed to “Barnes and his heirs.” Although both Butch and Jesse were married, their respective wives did not join in the deed. Barnes has been in continuous possession of the home since 1998. Samuel died on January 1, 2009 without issue, and Dixie is still alive.
In March 2009, Dixie, asserting that her title was held free of any claim by Barnes, instituted suit against Barnes to quiet title to Winterthur. Judgment should be for
(A) Barnes, because his prior recorded deed would be deemed to be outside Dixie’s chain of title.
(B) Barnes, because the devise to Dixie would be violative of the Rule against Perpetuities.
(C) Dixie, because she acquired fee simple interest at the time of Samuel’s death in January 2009.
(D) Dixie, because she acquired an indefeasible vested remainder under her father’s will in
1963.
- (C) In a suit to quiet title to the home, Dixie would prevail. At the time of Samuel’s death on January 1, 2009 (without issue), Dixie’s executory interest was executed into a legal estate in fee. In fact, Dixie’s interest vested automatically upon the Samuel’s death without issue, without any affirmative act on her part. Choice (A) is incorrect. Dixie is the holder of a shifting executory interest. When Samuel died without issue, title vested in Dixie. The issue in this question is whether or not Dixie takes as a future interest holder. Chain of title issues will arise when competing grantees are trying to acquire, title to the same present possessory estate. Dixie, as a future interest holder, has no one to compete with because as soon as Samuel died without issue, title was vested in her. Because there is no competition for which grantee holds title, chain of title is not in issue here. Choice (B) is incorrect. The Rule against Perpetuities would apply to Dixie’s shifting executory interest. Furthermore, Samuel will be a valid measuring life in this situation. Upon Samuel’s death, it will be known for certain whether the condition was met or not. Because Samuel died without issue, Dixie’s executory interest will vest immediately and will not violate the Rule against Perpetuities. Choice (D) is incorrect. A remainder is a future interest created in a third person which is intended to take afterthe naturaltermination of a preceding estate. However, Dixie does not have a remainder because she does not take after the natural termination of the preceding estate; rather, she takes only upon the happening (or non-happening) of a specified event. This is why her interest is best categorized as an executory interest.
- On January 1, a seller entered into an agreement of sale to convey his farm to a buyer. According to the contract, settlement (or closing) was to take place on March 1 of the same year.
On January 20, a barn on the property was destroyed by fire. When the buyer was notified of the loss the following day, he contacted the seller and told him that the deal was off because of the fire. As a result, the buyer filed suit against the seller on January 25 for a refund of his down payment of $5,000.
In his suit, the buyer will be unsuccessful and will be obliged to complete the contract if the court determines that on January 20
(A) the buyer held the legal title to the farm.
(B) the seller held the legal title to the farm.
(C) the buyer held the equitable title to the farm.
(D) the seller held the equitable title to the farm.
- (C) Equitable conversion applies when there is an enforceable obligation to sell land. The doctrine treats interests in land as if the land had already been converted into personal property. The doctrine states that when the sales contract is made, equity then considers the vendee as the owner of the land and the vendor as the owner of the purchase money. Applying the facts of the case to this doctrine, the buyer would be construed as the equitable owner of the land on January 20 when the barn was destroyed by fire. As a result, the buyer (as equitable owner) would become subject to all losses not occasioned by the fault of the seller. Therefore, if the court determined that the buyer held equitable title to the farm on January 20, he would be unsuccessful in his Lawsuit against the seller. Choice (A) is incorrect. Under equitable conversion the purchaser is regarded as the “equitable” owner of the land, although legal title remains with the seller. Under this doctrine, the seller would be the party to hold legal title, not the buyer. Because this choice misstates the type of title held by the buyer, it is incorrect. Choice (B) is incorrect. Under the doctrine of equitable conversion, the seller would be the party to hold legal title, not the buyer. However, this does not fully explain why the buyer would be unsuccessful in his suit and be required to complete the contract of sale. The most accurate reason why the buyer would be required to go through with the deal is that he held equitable title to the farm, making (C) the best choice. Choice (D) is incorrect. Under equitable conversion, the purchaser is regarded as the “equitable” owner of the land, although legal title remains with the seller. Under this doctrine, the seller would be the party to hold Legal title, not equitable title.
- An architect is the owner of a large two-story office building. The architect designed and supervised the building’s construction. He and his associate occupy the first floor for their architectural business. In June 2005, the architect entered into a valid and binding written lease with a tenant to rent the second floor office space for four years at a monthly rental of $5,000. The lease contained a provision wherein the architect was required to repaint the second floor premises after two years of occupancy.
On June 1, 2007, the tenant sent the architect a written letter requesting the repainting of the second-floor office space. The architect made no reply and failed to perform the repainting as the lease required. The cost of repainting the tenant’s office was estimated at $1,750. On July 1, 2008, the architect had still not repainted the premises. The following day, the tenant moved out, mailed the keys to the architect, and refused to pay any more rent. The lease was silent as to the rights and remedies of the tenant due to the architect’s failure to perform the repainting. There is no applicable statute in the jurisdiction.
In an appropriate action by the architect against the tenant for the rent due, the architect will
(A) win, because there was no constructive eviction.
(B) n, because the tenant had the remedy of self-help.
(C) lose, because he cannot maintain an action for rent while in breach of an express covenant.
(D) lose, because the obligation to pay rent is dependent on the architect’s performance of his express covenant.
- (A) At common law, because a tenant was deemed to receive an estate in land, his rights and duties were treated as independent of the Landlord’s rights and duties. Thus, if the landlord promised to keep the property in repair, a breach of this promise did not relieve the tenant from the duty of paying rent. According to the common law rationale, the rent was owed as payment for the estate, and the promise to do repairs was merely a colLateraL promise, which could be enforced only by a separate contractual suit brought by the tenant. This doctrine is generally referred to as the independence of covenants. Choice (B) is incorrect. (B) is wrong; the reason the Landlord will prevail is because the premises were not so uninhabitabLe (the basis for constructive eviction) as to force the tenant out and relieve the tenant of the duty to pay rent. This choice falsely suggests that the tenant had the remedy of self- help and failed to avail himself of that remedy by failing to perform the re-painting himself. In other words, self-help is not an appropriate remedy inasmuch as the tenant had no duty to make such repairs. Choice (C) is incorrect. Because the duty to pay rent was independent of the landlord’s rights and duties, a breach of the promise to keep the premises in repair still did not relieve the tenant from the duty of paying rent. This choice states that the landlord will lose in his action to collect rent because the architect cannot collect it while in breach of an express covenant. However, because the duty to pay rent is independent of the covenant to paint, the architect can sue for the rent and treat that as separate from his own failure to paint the premises. Because this choice states that a landlord cannot do both at the same time, it is incorrect. Choice (D) is incorrect. Because the duty to pay rent was independent of the landlord’s rights and duties, a breach of the promise to keep the premises in repair still did not relieve the tenant from the duty of paying rent. Because this choice states that the obligation to pay rent is dependent on the covenant to repair, it is incorrect.
- In a jurisdiction using the grantor—grantee indices, the following events have occurred in the order listed below:
(1) In 1962, a grantor conveyed to a man a tract of land by a deed that was immediately recorded.
(2) In 1982, the man died a widower and devised the tract of land to his assistant by a will that was duly admitted to probate.
(3) In 1993, the assistant mortgaged the tract of land to a mortgage company to secure a note for $25,000, due on December 31, 1993, with 9½ percent interest per annum.
(4) In 1996, the mortgage company recorded the mortgage.
(5) In 2003, in a signed writing, which the assistant delivered to his daughter, the assistant promised to convey the tract of land to the daughter by a quitclaim deed and pay the mortgage debt when it came due.
If the assistant refused to convey the tract of land to the daughter and the daughter brings suit against the assistant to compel a conveyance of the property, the daughter will most likely
(A) prevail, because there was an effective gift.
(B) prevail, because there was an effective assignment.
(C) not prevail, because there was no consideration to support the assistant’s promise to convey the property.
(D) not prevail, because specific performance will not be granted where there is an adequate remedy at law.
- (C) The statute of frauds requires a writing identifying the parties, describing the subject matter, stating the purchase price, and signed by the party to be charged. The writing the assistant deLivered to the daughter did not state any consideration —it was intended as a gift. Furthermore, it was not an effective gift because it was only a promise to convey. An effective gift requires a present transfer of an interest. If the transfer is to take effect in futuro, it is a mere promise to make a gift and unenforceable for lack of consideration. Smith and Boyer, Survey of the Law of Property, p. 469. Choice (A) is incorrect. There was no effective gift because this was a mere promise to make a gift. Moreover, this is unenforceable for lack of consideration. Because this choice states that the daughter would prevaiL, it is incorrect. Choice (B) is incorrect . This choice is incorrect because the assistant’s writing did not create an assignment. Choice (D) is incorrect. The daughter does not have an adequate remedy at law because there is a lack of consideration. Specific performance will not be granted due to a Lack of mutuality.